Buy BOOKS at Discounted Price

Functions

Class 12th Mathematics RD Sharma Volume 1 Solution
Exercise 2.1
  1. Which is one - one but not onto. Give an example of a function
  2. Which is not one - one but onto. Give an example of a function
  3. Which is neither one - one nor onto. Give an example of a function…
  4. f1 = {(1, 3), (2, 5), (3, 7)}; A = {1, 2, 3}, B = {3, 5, 7} Which of the…
  5. f2 = {(2, a), (3, b), (4, c)}; A = {2, 3, 4}, B = {a, b, c} Which of the…
  6. f3 = {(a, x), (b, x), (c, z), (d, z)}; A = {a, b, c, d}, B = {x, y, z} Which of…
  7. Prove that the function f : N → N, defined by f(x) = x^2 + x + 1 is one - one…
  8. Let A = {-1, 0, 1} and f = {(x, x^2) : x ∈ A}. Show that f : A → A is neither…
  9. f : N → N given by f(x) = x^2 Classify the following functions as injection,…
  10. f : Z → Z given by f(x) = x^2 Classify the following functions as injection,…
  11. f : N → N given by f(x) = x^3 Classify the following functions as injection,…
  12. f : Z → Z given by f(x) = x^3 Classify the following functions as injection,…
  13. f : R → R, defined by f(x) = |x| Classify the following functions as injection,…
  14. f : Z → Z, defined by f(x) = x^2 + x Classify the following functions as…
  15. f : Z → Z, defined by f(x) = x - 5 Classify the following functions as…
  16. f : R → R, defined by f(x) = sin x Classify the following functions as…
  17. f : R → R, defined by f(x) = x^3 + 1 Classify the following functions as…
  18. f : R → R, defined by f(x) = x^3 - x Classify the following functions as…
  19. f : R → R, defined by f(x) = sin^2 x + cos^2 x Classify the following functions…
  20. f: Q - {3} → Q, defined by f (x) = 2x+3/x-3 Classify the following functions as…
  21. f : Q → Q, defined by f(x) = x^3 + 1 Classify the following functions as…
  22. f : R → R, defined by f(x) = 5x^3 + 4 Classify the following functions as…
  23. f : R → R, defined by f(x) = 3 - 4x Classify the following functions as…
  24. f : R → R, defined by f(x) = 1 + x^2 Classify the following functions as…
  25. f: R → R, defined by f (x) = x/x^2 + 1 Classify the following functions as…
  26. If f: A → B is an injection such that range of f = {a}. Determine the number of…
  27. Show that the function f : R - {3} → R - {1} given by f (x) = x-2/x-3 is a…
  28. f (x) = x/2 Let A = [-1, 1], Then, discuss whether the following functions from…
  29. g(x) = |x| Let A = [-1, 1], Then, discuss whether the following functions from…
  30. h(x) = x^2 Let A = [-1, 1], Then, discuss whether the following functions from…
  31. {(x, y): x is a person, y is the mother of x} Are the following set of ordered…
  32. {(a, b) : a is a person, b is an ancestor of a} Are the following set of…
  33. Let A = {1, 2, 3}. Write all one - one from A to itself.
  34. If f : R → R be the function defined by f(x) = 4x^3 + 7, show that f is a…
  35. Show that the exponential function f: R → R, given by f(x) = ex, is one - one…
  36. Show that the logarithmic function f : R+^0 → R given by f(x) = loga x, a 0 is…
  37. If A = {1, 2, 3}, show that a one - one function f : A → A must be onto.…
  38. If A = {1, 2, 3}, show that an onto function f : A → A must be one - one.…
  39. Find the number of all onto functions from the set A = {1, 2, 3, …., n} to…
  40. Give examples of two one - one functions f1 and f2 from R to R such that f1 +…
  41. Give examples of two surjective function f1 and f2 from Z to Z such that f1 +…
  42. Show that if f1 and f2 are one - one maps from R to R, then the product f1 × f2…
  43. Suppose f1 and f2 are non - zero one - one functions from R to R. Is f_1/f_2…
  44. an injective map from A to B Given A = {2, 3, 4}, B = {2, 5, 6, 7}. Construct…
  45. a mapping from A to B which is not injective Given A = {2, 3, 4}, B = {2, 5,…
  46. a mapping from A to B. Given A = {2, 3, 4}, B = {2, 5, 6, 7}. Construct an…
  47. Show that f : R → R, given by f(x) = x - [x], is neither one - one nor onto.…
  48. Let f : N → N be defined by f (n) = cl n+1 , & n n-1 , & n Show that f is a…
Exercise 2.2
  1. f(x) = 2x + 3 and g(x) = x^2 + 5 Find gof and fog when f: R → R and g: R → R is…
  2. f(x) = 2x + x^2 and g(x) = x^3 Find gof and fog when f: R → R and g: R → R is…
  3. f(x) = x^2 + 8 and g(x) = 3x^3 + 1 Find gof and fog when f: R → R and g: R → R…
  4. f(x) = x and g(x) = |x| Find gof and fog when f: R → R and g: R → R is defined…
  5. f(x) = x^2 + 2x - 3 and g(x) = 3x - 4 Find gof and fog when f: R → R and g: R →…
  6. f(x) = 8x^3 and g(x) = x1/3 Find gof and fog when f: R → R and g: R → R is…
  7. Let f = {(3, 1), (9, 3), (12, 4)} and g = {(1, 3), (3, 3), (4, 9), (5, 9)}. Show…
  8. Let f = {(1, - 1), (4, - 2), (9, - 3), (16, 4)} and g = {(- 1, - 2), (- 2, - 4),…
  9. Let A = {a, b c}, B = {u, v, w} and let f and g be two functions from A to B and…
  10. Find fog (2) and gof (1) when: f: R → R; f(x) = x^2 + 8 and g: R → R; g(x) =…
  11. Let R+ be the set of all non - negative real numbers. If f: R+ → R+ and g: R+ →…
  12. Let f: R → R and g: R → R be defined by f(x) = x^2 and g(x) = x + 1. Show that…
  13. Let f: R → R an g: R → R be defined by f(x) = x + 1 and g(x) = x - 1. Show that…
  14. Verify associativity for the following three mappings: f: N → Z0 (the set of non…
  15. Consider f: N → N, g: N → N and h: N → R defined as f(x) = 2x, g(y) = 3y + 4…
  16. Give examples of two functions f: N → N and g: N → N such that gof is onto, but…
  17. Give examples of two functions f: N → Z and g: Z → Z such that gof is…
  18. If f: A → B and g: B → C are one - one functions show that gof is a one - one…
  19. If f: A → B and g: B → C are onto functions show that gof is an onto function.…
Exercise 2.3
  1. f(x) = ex, g(x) = logex Find fog and gof, if
  2. f(x) = x^2 , g(x) = cos x Find fog and gof, if
  3. f(x) = |x|, g(x) = sin x Find fog and gof, if
  4. f(x) = x + 1, g(x) = ex Find fog and gof, if
  5. f(x) = sin-1 x, g(x) = x^2 Find fog and gof, if
  6. f(x) = x + 1, g(x) = sin x Find fog and gof, if
  7. f(x) = x + 1, g(x) = 2x + 3 Find fog and gof, if
  8. f(x) = c, c ∈ R, g(x) = sin x^2 Find fog and gof, if
  9. f(x) = x^2 + 2, g (x) = 1 - 1/1-x Find fog and gof, if
  10. Let f(x) = x^2 + x + 1 and g(x) = sin x. Show that fog ≠ gof.
  11. If f(x) = |x|, prove that fof = f.
  12. If f(x) = 2x + 5 and g(x) = x^2 + 1 be two real functions, then describe each of…
  13. If f(x) = sin x and g(x) = 2x be two real functions, then describe gof and fog.…
  14. Let f, g, h be real functions given by f(x) = sin x, g(x) = 2x and h(x) = cos x.…
  15. Let f be any real function and let g be a function given by g(x) = 2x. Prove…
  16. If f (x) = root 1-x and g(x) = logex are two real functions, then describe…
  17. If f: (- pi /2 , pi /2) → R and g: [-1, 1] → R be defined as f(x) = tan x and g…
  18. If f (x) = root x+3 and g(x) = x^2 + 1 be two real functions, then find fog and…
  19. Let f be a real function given by f (x) = root x-2 . Find each of the…
  20. Let f be a real function given by f (x) = root x-2 . Find each of the…
  21. Let f be a real function given by f (x) = root x-2 . Find each of the…
  22. Let f be a real function given by f (x) = root x-2 . Find each of the…
  23. Let f (x) = ll 1+x , & 0 less than equal to x less than equal to 2 3-x , & 2x…
  24. If f, g: R → R be two functions defined as f(x) = |x| + x and g(x) = |x|-x for…
Exercise 2.4
  1. State with reasons whether the following functions have inverse: (i) f : [1, 2,…
  2. Find f-1 if it exists for f: A → B where (i) A = {0, -1, -3, 2}; B = {-9, -3, 0,…
  3. Consider f : {1, 2, 3} → {a, b, c} and g : {a, b, c} → {apple, ball, cat}…
  4. Let A = {1, 2, 3, 4}; B = {3, 5, 7, 9}; C = {7, 23, 47, 79} and f : A → B, g : B…
  5. Show that the function f : Q → Q defined by f(x) = 3x + 5 is invertible. Also,…
  6. Show that the function f : R → R defined by f(x) = 4x + 3 is invertible. Find…
  7. Consider f : R+→ [4, ∞) given by f(x) = x^2 + 4. Show that f is invertible with…
  8. If f (x) = 4x+3/6x-4 , x not equal 2/3 , show that (fof)(x) = x for all x not…
  9. Consider f : R+→ [-5, ∞) given by f(x) = 9x^2 + 6x - 5. Show that f is…
  10. If f : R → R be defined by f(x) = x^3 - 3, then prove that f-1 exists and find…
  11. A function f : R → R is defined as f(x) = x^3 + 4. Is it a bijection or not? In…
  12. If f : Q → Q, g : Q → Q are two functions defined by f(x) = 2x and g(x) = x +…
  13. Let A = R - {3} and B = R - {1}. Consider the function f : A → B defined by f…
  14. Consider the function f : R+→ [-9, ∞) given by f(x) = 5x^2 + 6x - 9. Prove that…
  15. Let f : N → N be a function defined as f(x) = 9x^2 + 6x - 5. Show that f : N →…
  16. Let f : R - - 4/3 → R be a function defined as f (x) = 4x/3x+4 . Show that f :…
  17. If f: R → (-1, 1) defined by f (x) = 10^x - 10^-x/10^x + 10^-x is invertible,…
  18. If f: R → (0, 2) defined by f (x) = e^x - e^-x/e^x + e^-x + 1 is invertible,…
  19. Let f : [-1, ∞) → [-1, ∞) is given by f(x) = (x + 1)^2 - 1. Show that f is…
  20. Let A = {x ϵ R | -1 ≤ x ≤ 1} and let f : A → A, g : A → A be two functions…
  21. Let f be a function from R to R such that f(x) = cos (x + 2). Is f invertible?…
  22. If A = {1, 2, 3, 4} and B = {a, b, c, d}, define any four bijections from A to…
  23. Let A and B be two sets each with finite number of elements. Assume that there…
  24. If f : A → A and g : A → A are two bijections, then prove that (i) fog is an…
Very Short Answer
  1. Which one of the following graphs represent a function?
  2. Which one of the following graphs represent a one-one function?
  3. If A = {1, 2, 3} and B = {a, b}, write total number of functions from A to B.…
  4. If A = {a, b, c} and B = {–2, –1, 0, 1, 2}, write total number of one-one functions…
  5. Write total number of one-one functions from set A = {1, 2, 3, 4} to set B = {a, b c}.…
  6. If f: R→ R is defined by f(x) = x2, write f–1(25).
  7. If f : C → C is defined by f(x) = x2, write f–1(–4). Here, C denotes the set of all…
  8. If f: R → R is given by f(x) = x3, write f–1(1).
  9. Let C denote the set of all complex numbers. A function f : C → C is defined by f(x) =…
  10. Let f be a function from C (set of all complex numbers) to itself given by f(x) = x3.…
  11. Let f : R → R be defined by f(x) = x4, write f–1(1).
  12. If f : C → C is defined by f(x) = x4, f–1(1).
  13. If f :R→R is defined by f(x) = x2, f–1(–25).
  14. If f : C → C is defined by f(x) = (x – 2)3, write f–1(–1).
  15. If f :R→R is defined by f(x) = 10x –7, then write f–1(x).
  16. Let f: ( - { pi }/{2} , frac { pi }/{2} ) arrowr be a function defined by f(x) =…
  17. If f : R → R defined by f(x) = 3x – 4 is invertible then write f–1(x).…
  18. If f : R → R, g : R → R are given by f(x) = (x + 1)2 and g(x) = x2 + 1, then write the…
  19. Let A = {x ϵR : –4 ≤ x ≤ x ≤ 4 and x ≠ 0} and f : A → R be defined by f (x) =…
  20. Let f:[ - { pi }/{2} , frac { pi }/{2} ] arrowa be defined by f(x) = sinx. If f is…
  21. Let f : R → R+ be defined by f(x) = ax, a 0 and a ≠ 1. Write f–1(x).…
  22. Let f : R – {–1} → R – {1} be given by f (x) = {x}/{x+1} Write f–1(x).…
  23. Let f:r - { - {3}/{5} } arrowr be a function defined as f (x) = {2x}/{5x+3}…
  24. Let f : R → R, g : R → R be two functions defined by f(x) = x2 + x + 1 and g(x) = 1 –…
  25. Let f : R →R be defined as f (x) = {2x-3}/{4} Write fof–1(1).…
  26. Let f be an invertible real function. Write (f–1 of) (1) + (f–1 of) (2) + … +(f–1 of)…
  27. Let A = {1, 2, 3, 4} and B = {a, b} be two sets. Write total number of onto functions…
  28. Write the domain of the real function f (x) = root {x-[x]}
  29. Write the domain of the real function f (x) = {1}/{ root {|x|-x} }…
  30. Write whether f : R → R given by f (x) = x + root { x^{2} } is one-one, many-one,…
  31. If f(x) = x + 7 and g(x) = x – 7, x ϵ R, write fog(7).
  32. What is the range of the function f (x) = {|x-1|}/{x-1}?
  33. If f : R → R be defined by f(x) = (3 – x3)1/3, then find fof(x).
  34. If f : R → R is defined by f(x) = 3x + 2, find f(f (x)).
  35. Let A = {1, 2, 3}, B = {4, 5, 6, 7} and let f = {(1, 4), (2, 5), (3, 6)} be a function…
  36. If f : {5, 6} → {2, 3} and g : {2, 3} → {5, 6} are given by f = {(5, 2), (6, 3)} and g…
  37. Let f : R → R be the function defined by f(x) = 4x – 3 for all x ϵ R. Then write f–1.…
  38. Which one the following relations on A = {1, 2, 3} is a function?f = {(1, 3), (2, 3),…
  39. Write the domain of the real function f defined by f (x) = root { 25-x^{2} }…
  40. Let A = {a, b, c, d} and f : A → A be given by f = {(a, b), (b, d), (c, a), (d, c)},…
  41. Let f, g : R → R be defined by f(x) = 2x + 1 and g(x) = x2 – 2 for all x ϵ R,…
  42. If the mapping f : {1, 3, 4} → {1, 2, 5} and g : {1, 2, 5} → {1, 3}, given byf = {(1,…
  43. If a function g = {(1, 1), (2, 3), (3, 5), (4, 7)} is described by g(x) = αx + β, find…
  44. If f(x) = 4 – (x – 7)3, write f–1(x).
  45. Q129 Write the domain of the real function f (x) = root {[x]-x}
Mcq
  1. Let A = {x ϵ R : –1 ≤ x ≤ 1} = B and C = {x ϵ R : X ≥ 0} and let S = {(x, y) ϵ A × B :…
  2. f: R → R given by f (x) = x + root { x^{2} } is Mark the correct alternative in each…
  3. If f : A → B given by 3f(x) + 2–x = 4 is a bijection, then Mark the correct alternative…
  4. The function f : R → R defined by f(x) = 2x + 2|x| is Mark the correct alternative in…
  5. Let the function f : R – {–b} → R – {1} be defined by f (x) = {x+a}/{x+b} , a not…
  6. The function f : A → B defined by f(x) = –x2 + 6x – 8 is a bijection, if Mark the…
  7. Let A = {x ϵR : –1 ≤ x ≤ 1} = B. Then, the mapping f : A → B given by f(x) = x |x| is…
  8. Let f: R → R be given by f(x) = [x]2 + [x + 1]–3, where [x] denotes the greatest…
  9. Let M be the set of all 2 × 2 matrices with entries from the set R of real numbers.…
  10. The function f : [0, ∞) → R given by f(x) f (x) = {x}/{x+1} is Mark the correct…
  11. The range of the function f(x) = 7–xPx–3 is Mark the correct alternative in each of…
  12. A function f from the set on natural numbers to integers defined by. f (n) = { {ll} {…
  13. Let f be an injective map with domain {x, y, z} and range {1, 2, 3} such that exactly…
  14. Which of the following functions from Z to itself are bijections? Mark the correct…
  15. Which of the following functions from A = {x : –1 ≤ x ≤ 1} to itself are bijections?…
  16. Let A = {x : –1 ≤ x ≤ 1} and f : A → A such that f(x) = x |x|, then f is Mark the…
  17. If the function f: R → A given by f (x) = { x^{2} }/{ x^{2} + 1 } is a surjection,…
  18. If a function f : [2, ∞) → B defined by f(x) = x2 – 4x + 5 is a bijection, then B =…
  19. The function f : R → R defined by f(x) = (x – 1) (x – 2) (x – 3) is Mark the correct…
  20. The function f : [–1/2, 1/2] → [π/2, π/2] defined by f(x) = sin–1 (3x – 4x3) is Mark…
  21. Let f : R → R be a function defined by f (x) = { e^{|x|}-e^{-x} }/{ e^{x} + e^{-x}…
  22. Let f : R –{n} → R be a function defined by f (x) = {x-m}/{x-1} where m ≠ n. Then,…
  23. Let f : R → R be a function defined by f (x) = { x^{2} - 8 }/{ x^{2} + 2 } Then, f…
  24. f : R → R is defined by f (x) = { e^ { x^{2} } - e^ { - x^{2} } }/{ e^ { x^{2} } +…
  25. The function f : R → R, f(x) = x2 is Mark the correct alternative in each of the…
  26. A function f from the set of natural, numbers to the set of integers defined by f (n)…
  27. Which of the following functions from A = {x ϵ R : –1 ≤ x ≤ 1} to itself are…
  28. Let f: Z → Z be given by f (x) = { {ll} { {x}/{2} , } & { x } { 0 , } & { x }…
  29. The function f : R → R defined by f(x) = 6x + 6|x| is Mark the correct alternative in…
  30. Let f(x) = x2 and g(x) = 2x. Then the solution set of the equation fog(x) = gof (x) is…
  31. If f: R → R is given by f(x) = 3x – 5, then f–1(x) Mark the correct alternative in…
  32. If g(f(x)) = |sin x| and f (g(x)) = ( sinroot {x} ) ^{2} , then Mark the correct…
  33. The inverse of the function f : R → [x ϵ R : x 1] given by f (x) = { e^{x} -…
  34. Let A = {x ϵR : x ≥ 1}. The inverse of the function f : A → A given by f(x) = 2x(x–1),…
  35. Let A = {x ϵ R: x ≤ 1} and f : A → A given by f(x) = x(2 – x).Then, f–1(x) is Mark the…
  36. Let f (x) = {1}/{1-x} Then, {fo(fof)} (x) Mark the correct alternative in each of…
  37. If the function f: R → R be such that f(x) = x – [x], where [x] denotes the greatest…
  38. If F: [1, ∞) → [2, ∞) is given by f (x) = x + {1}/{x} then f–1(x) equals. Mark the…
  39. Let g(x) = 1 + x – [x] and f (x) = { {ll} { - 1 , } &{x0} where [x] denotes the…
  40. Let f (x) = {dx}/{x+1} , x not equal -1 Then, for what value of α is f(f(x)) = x?…
  41. The distinct linear functions which map [–1, 1] onto [0, 2] are Mark the correct…
  42. Let f : [2, ∞) → X be defined by f(x) = 4x – x2. Then, f is invertible, if X = Mark…
  43. If f : R → (–1, 1) is defined by f (x) = {-x|x|}/{ 1+x^{2} } then f–1(x) equals…
  44. Let [x] denote the greatest integer less than or equal to x. If f(x) = sin–1 x, g(x) =…
  45. If g(x) = x2 + x – 2 and {1}/{2} gof (x) = 2x2 – 5x + 2, then f(x) is equal to…
  46. If f(x) = sin2 x and the composite function g(f(x)) = |sin x|, then g(x) is equal to…
  47. If f : R → R is given by f(x) = x3 + 3, then f–1(x) is equal to Mark the correct…
  48. Let f(x) = x3 be a function with domain {0, 1, 2, 3}. Then domain of f–1 is Mark the…
  49. Let f : R → R be given by f(x) = x2 – 3. Then, f–1 is given by Mark the correct…
  50. Let f : R → R be given by f(x) = tan x. Then, f–1(1) is Mark the correct alternative…
  51. Let f: R → R be defined as f (x) = { {c} { 2x , x>3 } { x^{2} , 1Then, find f(–1) +…
  52. Let A = {1, 2, …, n} and B = {a, b}. Then the number of subjections from A into B is…
  53. If the set A contains 5 elements and the set B contains 6 elements, then the number of…
  54. If the set A contains 7 elements and the set B contains 10 elements, then the number…
  55. Let f:r - { {3}/{5} } arrowr be defined by f (x) = {3x+2}/{5x-3} Then, Mark the…

Exercise 2.1
Question 1.

Give an example of a function

Which is one – one but not onto.


Answer:

TIP:One – One Function: – A function is said to be a one – one functions or an injection if different elements of A have different images in B.


So, is One – One function


⇔ a≠b


⇒ f(a)≠f(b) for all


⇔ f(a) = f(b)


⇒ a = b for all


Onto Function: – A function is said to be a onto function or surjection if every element of A i.e, if f(A) = B or range of f is the co – domain of f.


So, is Surjection iff for each , there exists such that f(a) = b


Now, Let, given by f(x) = x2


Check for Injectivity:


Let x,y be elements belongs to N i.e such that


So, from definition


⇒ f(x) = f(y)


⇒ x2 = y2


⇒ x2 – y2 = 0


⇒ (x – y)(x + y) = 0


As therefore x + y>0


⇒ x – y = 0


⇒ x = y


Hence f is One – One function


Check for Surjectivity:


Let y be element belongs to N i.e be arbitrary, then


⇒ f(x) = y


⇒ x2 = y



not belongs to N for non–perfect square value of y.


Therefore no non – perfect square value of y has a pre image in domain N.


Hence, given by f(x) = x2 is One – One but not onto.



Question 2.

Give an example of a function

Which is not one – one but onto.


Answer:

TIP:One – One Function: – A function is said to be a one – one functions or an injection if different elements of A have different images in B.


So, is One – One function


⇔ a≠b


⇒ f(a)≠f(b) for all


⇔ f(a) = f(b)


⇒ a = b for all


Onto Function: – A function is said to be a onto function or surjection if every element of A i.e, if f(A) = B or range of f is the co – domain of f.


So, is Surjection iff for each , there exists such that f(a) = b


Now, Let, given by f(x) = x3 – x


Check for Injectivity:


Let x,y be elements belongs to R i.e such that


So, from definition


⇒ f(x) = f(y)


⇒ x3 – x = y3 – y


⇒ x3 – y3 – (x – y) = 0


⇒ (x – y)(x2 + xy + y2 – 1) = 0


As x2 + xy + y2 ≥ 0


⇒ therefore x2 + xy + y2 – 1≥ – 1


⇒ x – y≠0


⇒ x ≠ y for some


Hence f is not One – One function


Check for Surjectivity:


Let y be element belongs to R i.e be arbitrary, then


⇒ f(x) = y


⇒ x3 – x = y


⇒ x3 – x – y = 0


Now, we know that for 3 degree equation has a real root


So, let be that root




Thus for clearly , there exist such that f(x) = y


Therefore f is onto


⇒ Hence, given by f(x) = x3 – x is not One – One but onto



Question 3.

Give an example of a function

Which is neither one – one nor onto.


Answer:

TIP:One – One Function: – A function is said to be a one – one functions or an injection if different elements of A have different images in B.


So, is One – One function


⇔ a≠b


⇒ f(a)≠f(b) for all


⇔ f(a) = f(b)


⇒ a = b for all


Onto Function: – A function is said to be a onto function or surjection if every element of A i.e, if f(A) = B or range of f is the co – domain of f.


So, is Surjection iff for each , there exists such that f(a) = b


Now, Let, given by f(x) = 5


As we know


A constant function is neither one – one nor onto.


So, here f(x) = 5 is constant function


Therefore


given by f(x) = 5 is neither one – one nor onto function.



Question 4.

Which of the following functions from A to B are one – one and onto?

f1 = {(1, 3), (2, 5), (3, 7)}; A = {1, 2, 3}, B = {3, 5, 7}


Answer:

TIP:One – One Function: – A function is said to be a one – one functions or an injection if different elements of A have different images in B.


So, is One – One function


⇔ a≠b


⇒ f(a)≠f(b) for all


⇔ f(a) = f(b)


⇒ a = b for all


Onto Function: – A function is said to be a onto function or surjection if every element of A i.e, if f(A) = B or range of f is the co – domain of f.


So, is Surjection iff for each , there exists such that f(a) = b


Now, As given,


f1 = {(1, 3), (2, 5), (3, 7)}


A = {1, 2, 3}, B = {3, 5, 7}


Thus we can see that,


Check for Injectivity:


Every element of A has a different image from B


Hence f is a One – One function


Check for Surjectivity:


Also, each element of B is an image of some element of A


Hence f is Onto.



Question 5.

Which of the following functions from A to B are one – one and onto?

f2 = {(2, a), (3, b), (4, c)}; A = {2, 3, 4}, B = {a, b, c}


Answer:

TIP:One – One Function: – A function is said to be a one – one functions or an injection if different elements of A have different images in B.


So, is One – One function


⇔ a≠b


⇒ f(a)≠f(b) for all


⇔ f(a) = f(b)


⇒ a = b for all


Onto Function: – A function is said to be a onto function or surjection if every element of A i.e, if f(A) = B or range of f is the co – domain of f.


So, is Surjection iff for each , there exists such that f(a) = b


Now, As given,


f2 = {(2, a), (3, b), (4, c)}


A = {2, 3, 4}, B = {a, b, c}


Thus we can see that


Check for Injectivity:


Every element of A has a different image from B


Hence f is a One – One function


Check for Surjectivity:


Also, each element of B is an image of some element of A


Hence f is Onto.



Question 6.

Which of the following functions from A to B are one – one and onto?

f3 = {(a, x), (b, x), (c, z), (d, z)}; A = {a, b, c, d}, B = {x, y, z}


Answer:

TIP:One – One Function: – A function is said to be a one – one functions or an injection if different elements of A have different images in B.


So, is One – One function


⇔ a≠b


⇒ f(a)≠f(b) for all


⇔ f(a) = f(b)


⇒ a = b for all


Onto Function: – A function is said to be a onto function or surjection if every element of A i.e, if f(A) = B or range of f is the co – domain of f.


So, is Surjection iff for each , there exists such that f(a) = b


Now, As given,


f3 = {(a, x), (b, x), (c, z), (d, z)}


A = {a, b, c, d}, B = {x, y, z}


Thus we can clearly see that


Check for Injectivity:


Every element of A does not have different image from B


Since,


f3(a) = x = f3(b) and f3(c) = z = f3(d)


Therefore f is not One – One function


Check for Surjectivity:


Also each element of B is not image of any element of A


Hence f is not Onto.



Question 7.

Prove that the function f : N → N, defined by f(x) = x2 + x + 1 is one – one but not onto.


Answer:

TIP:One – One Function: – A function is said to be a one – one functions or an injection if different elements of A have different images in B.


So, is One – One function


⇔ a≠b


⇒ f(a)≠f(b) for all


⇔ f(a) = f(b)


⇒ a = b for all


Onto Function: – A function is said to be a onto function or surjection if every element of A i.e, if f(A) = B or range of f is the co – domain of f.


So, is Surjection iff for each , there exists such that f(a) = b


Now, given by f(x) = x2 + x + 1


Check for Injectivity:


Let x,y be elements belongs to N i.e such that


So, from definition


⇒ f(x) = f(y)


⇒ x2 + x + 1 = y2 + y + 1


⇒ x2 – y2 + x – y = 0


⇒ ( x – y )( x + y + 1) = 0


As therefore x + y + 1>0


⇒ x – y = 0


⇒ x = y


Hence f is One – One function


Check for Surjectivity:


y be element belongs to N i.e be arbitrary


Since for y > 1, we do not have any pre image in domain N.


Hence, f is not Onto function.



Question 8.

Let A = {–1, 0, 1} and f = {(x, x2) : x ∈ A}. Show that f : A → A is neither one – one nor onto.


Answer:

TIP:One – One Function: – A function is said to be a one – one functions or an injection if different elements of A have different images in B.


So, is One – One function


⇔ a≠b


⇒ f(a)≠f(b) for all


⇔ f(a) = f(b)


⇒ a = b for all


Onto Function: – A function is said to be a onto function or surjection if every element of A i.e, if f(A) = B or range of f is the co – domain of f.


So, is Surjection iff for each , there exists such that f(a) = b


Now, We have, A = {–1, 0, 1} and f = {(x, x2) : x ∈ A}.


To Prove: – f : A → A is neither One – One nor onto function


Check for Injectivity:


We can clearly see that


f(1) = 1


and f( – 1) = 1


Therefore


f(1) = f( – 1)


⇒ Every element of A does not have different image from A


Hence f is not One – One function


Check for Surjectivity:


Since, y = – 1 be element belongs to A


i.e in co – domain does not have any pre image in domain A.


Hence, f is not Onto function.



Question 9.

Classify the following functions as injection, surjection or bijection:

f : N → N given by f(x) = x2


Answer:

TIP:One – One Function: – A function is said to be a one – one functions or an injection if different elements of A have different images in B.


So, is One – One function


⇔ a≠b


⇒ f(a)≠f(b) for all


⇔ f(a) = f(b)


⇒ a = b for all


Onto Function: – A function is said to be a onto function or surjection if every element of A i.e, if f(A) = B or range of f is the co – domain of f.


So, is Surjection iff for each , there exists such that f(a) = b


Bijection Function: – A function is said to be a bijection function if it is one – one as well as onto function.


Now, given by f(x) = x2


Check for Injectivity:


Let x,y be elements belongs to N i.e such that


So, from definition


⇒ f(x) = f(y)


⇒ x2 = y2


⇒ x2 – y2 = 0


⇒ (x – y)(x + y) = 0


As therefore x + y>0


⇒ x – y = 0


⇒ x = y


Hence f is One – One function


Check for Surjectivity:


Let y be element belongs to N i.e be arbitrary, then


⇒ f(x) = y


⇒ x2 = y



not belongs to N for non–perfect square value of y.


Therefore no non – perfect square value of y has a pre–image in domain N.


Hence, f is not Onto function.


Thus, Not Bijective also.



Question 10.

Classify the following functions as injection, surjection or bijection:

f : Z → Z given by f(x) = x2


Answer:

TIP:One – One Function: – A function is said to be a one – one functions or an injection if different elements of A have different images in B.


So, is One – One function


⇔ a≠b


⇒ f(a)≠f(b) for all


⇔ f(a) = f(b)


⇒ a = b for all


Onto Function: – A function is said to be a onto function or surjection if every element of A i.e, if f(A) = B or range of f is the co – domain of f.


So, is Surjection iff for each , there exists such that f(a) = b


Bijection Function: – A function is said to be a bijection function if it is one – one as well as onto function.


Now, f : Z → Z given by f(x) = x2


Check for Injectivity:


Let x1, – x1 be elements belongs to Z i.e such that


So, from definition


⇒ x1 ≠ – x1


⇒ (x1)2 = ( – x1)2


⇒ f(x1)2 = f( – x1)2


Hence f is not One – One function


Check for Surjectivity:


Let y be element belongs to Z i.e be arbitrary, then


⇒ f(x) = y


⇒ x2 = y



not belongs to Z for non–perfect square value of y.


Therefore no non – perfect square value of y has a pre–image in domain Z.


Hence, f is not Onto function.


Thus, Not Bijective also



Question 11.

Classify the following functions as injection, surjection or bijection:

f : N → N given by f(x) = x3


Answer:

TIP:One – One Function: – A function is said to be a one – one functions or an injection if different elements of A have different images in B.


So, is One – One function


⇔ a≠b


⇒ f(a)≠f(b) for all


⇔ f(a) = f(b)


⇒ a = b for all


Onto Function: – A function is said to be a onto function or surjection if every element of A i.e, if f(A) = B or range of f is the co – domain of f.


So, is Surjection iff for each , there exists such that f(a) = b


Bijection Function: – A function is said to be a bijection function if it is one – one as well as onto function.


Now, f : N → N given by f(x) = x3


Check for Injectivity:


Let x,y be elements belongs to N i.e such that


⇒ f(x) = f(y)


⇒ x3 = y3


⇒ x3 – y3 = 0


⇒ (x – y)(x2 + y2 + xy) = 0


As therefore x2 + y2 + xy >0


⇒ x – y = 0


⇒ x = y


Hence f is One – One function


Check for Surjectivity:


Let y be element belongs to N i.e be arbitrary, then


⇒ f(x) = y


⇒ x3 = y



not belongs to N for non–perfect cube value of y.


Since f attain only cubic number like 1,8,27….,


Therefore no non – perfect cubic values of y in N (co – domain) has a pre–image in domain N.


Hence, f is not onto function


Thus, Not Bijective also



Question 12.

Classify the following functions as injection, surjection or bijection:

f : Z → Z given by f(x) = x3


Answer:

TIP:One – One Function: – A function is said to be a one – one functions or an injection if different elements of A have different images in B.


So, is One – One function


⇔ a≠b


⇒ f(a)≠f(b) for all


⇔ f(a) = f(b)


⇒ a = b for all


Onto Function: – A function is said to be a onto function or surjection if every element of A i.e, if f(A) = B or range of f is the co – domain of f.


So, is Surjection iff for each , there exists such that f(a) = b


Bijection Function: – A function is said to be a bijection function if it is one – one as well as onto function.


Now, f : Z → Z given by f(x) = x3


Check for Injectivity:


Let x,y be elements belongs to Z i.e such that


⇒ f(x) = f(y)


⇒ x3 = y3


⇒ x3 – y3 = 0


⇒ x = y


Hence f is One – One function


Check for Surjectivity:


Let y be element belongs to Z i.e be arbitrary, then


⇒ f(x) = y


⇒ x3 = y




Since f attain only cubic number like 1,8,27….


Therefore no non – perfect cubic values of y in Z (co – domain) have a pre–image in domain Z.


Hence, f is not onto function


Thus, Not Bijective also



Question 13.

Classify the following functions as injection, surjection or bijection:

f : R → R, defined by f(x) = |x|


Answer:

TIP:One – One Function: – A function is said to be a one – one functions or an injection if different elements of A have different images in B.


So, is One – One function


⇔ a≠b


⇒ f(a)≠f(b) for all


⇔ f(a) = f(b)


⇒ a = b for all


Onto Function: – A function is said to be a onto function or surjection if every element of A i.e, if f(A) = B or range of f is the co – domain of f.


So, is Surjection iff for each , there exists such that f(a) = b


Bijection Function: – A function is said to be a bijection function if it is one – one as well as onto function.


Now, f : R → R, defined by f(x) = |x|


Check for Injectivity:


Let x,y be elements belongs to R i.e such that


Case i


⇒ x = y


⇒ |x| = |y|


Case ii


⇒ – x = y


⇒ | – x| = |y|


⇒ x = |y|


Hence from case i and case ii f is not One – One function


Check for Surjectivity:


Since f attain only positive values, for negative real numbers in R


(co – domain) there is no pre–image in domain R.


Hence, f is not onto function


Thus, Not Bijective also



Question 14.

Classify the following functions as injection, surjection or bijection:

f : Z → Z, defined by f(x) = x2 + x


Answer:

TIP:One – One Function: – A function is said to be a one – one functions or an injection if different elements of A have different images in B.


So, is One – One function


⇔ a≠b


⇒ f(a)≠f(b) for all


⇔ f(a) = f(b)


⇒ a = b for all


Onto Function: – A function is said to be a onto function or surjection if every element of A i.e, if f(A) = B or range of f is the co – domain of f.


So, is Surjection iff for each , there exists such that f(a) = b


Bijection Function: – A function is said to be a bijection function if it is one – one as well as onto function.


Now, f : Z → Z given by f(x) = x2 + x


Check for Injectivity:


Let x,y be elements belongs to Z i.e such that


⇒ f(x) = f(y)


⇒ x2 + x = y2 + y


⇒ x2 – y2 + x – y = 0


⇒ (x – y)( x + y + 1) = 0


Either (x – y) = 0 or ( x + y + 1) = 0


Case i :


If x – y = 0


⇒ x = y


Hence f is One – One function


Case ii :


If x + y + 1 = 0


⇒ x + y = – 1


⇒ x ≠ y


Hence f is not One – One function


Thus from case i and case ii f is not One – One function


Check for Surjectivity:


As


Let x be element belongs to Z i.e be arbitrary, then


⇒ f(x) = 1


⇒ x2 + x = 1


⇒ x2 + x – 1 = 0



Above value of x does not belong to Z


Therefore no values of x in Z (co – domain) have a pre–image in domain Z.


Hence, f is not onto function


Thus, Not Bijective also



Question 15.

Classify the following functions as injection, surjection or bijection:

f : Z → Z, defined by f(x) = x – 5


Answer:

TIP:One – One Function: – A function is said to be a one – one functions or an injection if different elements of A have different images in B.


So, is One – One function


⇔ a≠b


⇒ f(a)≠f(b) for all


⇔ f(a) = f(b)


⇒ a = b for all


Onto Function: – A function is said to be a onto function or surjection if every element of A i.e, if f(A) = B or range of f is the co – domain of f.


So, is Surjection iff for each , there exists such that f(a) = b


Bijection Function: – A function is said to be a bijection function if it is one – one as well as onto function.


Now, f : Z → Z given by f(x) = x – 5


Check for Injectivity:


Let x,y be elements belongs to Z i.e such that


⇒ f(x) = f(y)


⇒ x – 5 = y – 5


⇒ x = y


Hence, f is One – One function


Check for Surjectivity:


Let y be element belongs to Z i.e be arbitrary, then


⇒ f(x) = y


⇒ x – 5 = y


⇒ x = y + 5


Above value of x belongs to Z


Therefore for each element in Z (co – domain) there exists an element in domain Z.


Hence, f is onto function


Thus, Bijective function



Question 16.

Classify the following functions as injection, surjection or bijection:

f : R → R, defined by f(x) = sin x


Answer:

TIP:One – One Function: – A function is said to be a one – one functions or an injection if different elements of A have different images in B.


So, is One – One function


⇔ a≠b


⇒ f(a)≠f(b) for all


⇔ f(a) = f(b)


⇒ a = b for all


Onto Function: – A function is said to be a onto function or surjection if every element of A i.e, if f(A) = B or range of f is the co – domain of f.


So, is Surjection iff for each , there exists such that f(a) = b


Bijection Function: – A function is said to be a bijection function if it is one – one as well as onto function.


Now, f : R → R, defined by f(x) = sin x


Check for Injectivity:


Let x,y be elements belongs to R i.e such that


⇒ f(x) = f(y)


⇒ sin x = sin y



⇒ x ≠ y


Hence, f is not One – One function


Check for Surjectivity:


Let y be element belongs to R i.e be arbitrary, then


⇒ f(x) = y


⇒ sin x = y



Now, for y>1 x not belongs to R (Domain)


Hence, f is not onto function


Thus, It is also not Bijective function



Question 17.

Classify the following functions as injection, surjection or bijection:

f : R → R, defined by f(x) = x3 + 1


Answer:

TIP:One – One Function: – A function is said to be a one – one functions or an injection if different elements of A have different images in B.


So, is One – One function


⇔ a≠b


⇒ f(a)≠f(b) for all


⇔ f(a) = f(b)


⇒ a = b for all


Onto Function: – A function is said to be a onto function or surjection if every element of A i.e, if f(A) = B or range of f is the co – domain of f.


So, is Surjection iff for each , there exists such that f(a) = b


Bijection Function: – A function is said to be a bijection function if it is one – one as well as onto function.


Now, Let, given by f(x) = x3 + 1


Check for Injectivity:


Let x,y be elements belongs to R i.e such that


So, from definition


⇒ f(x) = f(y)


⇒ x3 + 1 = y3 + 1


⇒ x3 = y3


⇒ x = y


Hence f is One – One function


Check for Surjectivity:


Let y be element belongs to R i.e be arbitrary, then


⇒ f(x) = y


⇒ x3 + 1 = y


Now, we know that for 3 degree equation has a real root


So, let be that root




Thus for clearly , there exist such that f(x) = y


Therefore f is onto


Thus, It is also Bijective function



Question 18.

Classify the following functions as injection, surjection or bijection:

f : R → R, defined by f(x) = x3 – x


Answer:

TIP:One – One Function: – A function is said to be a one – one functions or an injection if different elements of A have different images in B.


So, is One – One function


⇔ a≠b


⇒ f(a)≠f(b) for all


⇔ f(a) = f(b)


⇒ a = b for all


Onto Function: – A function is said to be a onto function or surjection if every element of A i.e, if f(A) = B or range of f is the co – domain of f.


So, is Surjection iff for each , there exists such that f(a) = b


Bijection Function: – A function is said to be a bijection function if it is one – one as well as onto function.


Now, Let, given by f(x) = x3 + x


Check for Injectivity:


Let x,y be elements belongs to R i.e such that


So, from definition


⇒ f(x) = f(y)


⇒ x3 – x = y3 – y


⇒ x3 – y3 – (x – y) = 0


⇒ (x – y)(x2 + xy + y2 – 1) = 0


Hence f is not One – One function


Check for Surjectivity:


Let y be element belongs to R i.e be arbitrary, then


⇒ f(x) = y


⇒ x3 – x = y


⇒ x3 – x – y = 0


Now, we know that for 3 degree equation has a real root


So, let be that root




Thus for clearly , there exist such that f(x) = y


Therefore f is onto


Thus, It is not Bijective function



Question 19.

Classify the following functions as injection, surjection or bijection:

f : R → R, defined by f(x) = sin2x + cos2x


Answer:

TIP:One – One Function: – A function is said to be a one – one functions or an injection if different elements of A have different images in B.


So, is One – One function


⇔ a≠b


⇒ f(a)≠f(b) for all


⇔ f(a) = f(b)


⇒ a = b for all


Onto Function: – A function is said to be a onto function or surjection if every element of A i.e, if f(A) = B or range of f is the co – domain of f.


So, is Surjection iff for each , there exists such that f(a) = b


Bijection Function: – A function is said to be a bijection function if it is one – one as well as onto function.


Now, f : R → R, defined by f(x) = sin2x + cos2x


Check for Injectivity and Check for Surjectivity


Let x be element belongs to R i.e such that


So, from definition


⇒ f(x) = sin2x + cos2x


⇒ f(x) = sin2x + cos2x


⇒ f(x) = 1


⇒ f(x) = constant


We know that a constant function is neither One – One function nor onto function.


Thus, It is not Bijective function



Question 20.

Classify the following functions as injection, surjection or bijection:

f: Q – {3} → Q, defined by


Answer:

TIP:One – One Function: – A function is said to be a one – one functions or an injection if different elements of A have different images in B.


So, is One – One function


⇔ a≠b


⇒ f(a)≠f(b) for all


⇔ f(a) = f(b)


⇒ a = b for all


Onto Function: – A function is said to be a onto function or surjection if every element of A i.e, if f(A) = B or range of f is the co – domain of f.


So, is Surjection iff for each , there exists such that f(a) = b


Bijection Function: – A function is said to be a bijection function if it is one – one as well as onto function.


Now, f : R → R given by


Check for Injectivity:


Let x,y be elements belongs to Q i.e such that


⇒ f(x) = f(y)



⇒ (2x + 3)(y – 3) = (2y + 3)(x – 3)


⇒ 2xy – 6x + 3y – 9 = 2xy – 6y + 3x – 9


⇒ – 6x + 3y = – 6y + 3x


⇒ – 6x + 3y + 6y – 3x = 0


⇒ – 9x + 9y = 0


⇒ x = y


Thus, f is One – One function


Check for Surjectivity:


Let y be element belongs to Q i.e be arbitrary, then


⇒ f(x) = y



⇒ 2x + 3 = y (x – 3)


⇒ 2x + 3 = xy – 3y


⇒ 2x – xy = – 3(y + 1)



Above value of x belongs to Q – [3] for y = 2


Therefore for each element in Q – [3] (co – domain), there does not exist an element in domain Q.


Hence, f is not onto function


Thus, Not Bijective function



Question 21.

Classify the following functions as injection, surjection or bijection:

f : Q → Q, defined by f(x) = x3 + 1


Answer:

TIP:One – One Function: – A function is said to be a one – one functions or an injection if different elements of A have different images in B.


So, is One – One function


⇔ a≠b


⇒ f(a)≠f(b) for all


⇔ f(a) = f(b)


⇒ a = b for all


Onto Function: – A function is said to be a onto function or surjection if every element of A i.e, if f(A) = B or range of f is the co – domain of f.


So, is Surjection iff for each , there exists such that f(a) = b


Bijection Function: – A function is said to be a bijection function if it is one – one as well as onto function.


Now, f : Q → Q, defined by f(x) = x3 + 1


Check for Injectivity:


Let x,y be elements belongs to Q i.e such that


⇒ f(x) = f(y)


⇒ x3 + 1 = y3 + 1


⇒ x3 = y3


⇒ x = y


Hence, f is One – One function


Check for Surjectivity:


Let y be element belongs to Q i.e be arbitrary, then


⇒ x3 + 1 = y


⇒ x3 + 1 – y = 0


Now, we know that for 3 degree equation has a real root


So, let be that root




Thus for clearly , there exist such that f(x) = y


Therefore f is onto


Thus, It is a Bijective function



Question 22.

Classify the following functions as injection, surjection or bijection:

f : R → R, defined by f(x) = 5x3 + 4


Answer:

TIP:One – One Function: – A function is said to be a one – one functions or an injection if different elements of A have different images in B.


So, is One – One function


⇔ a≠b


⇒ f(a)≠f(b) for all


⇔ f(a) = f(b)


⇒ a = b for all


Onto Function: – A function is said to be a onto function or surjection if every element of A i.e, if f(A) = B or range of f is the co – domain of f.


So, is Surjection iff for each , there exists such that f(a) = b


Bijection Function: – A function is said to be a bijection function if it is one – one as well as onto function.


Now, f : R → R, defined by f(x) = 5x3 + 4


Check for Injectivity:


Let x,y be elements belongs to R i.e such that


⇒ f(x) = f(y)


⇒ 5x3 + 4 = 5y3 + 4


⇒ x3 = y3


⇒ x = y


Hence, f is One – One function


Check for Surjectivity:


Let y be element belongs to R i.e be arbitrary, then


⇒ 5x3 + 4 = y


⇒ 5x3 + 4 – y = 0


Now, we know that for 3 degree equation has a real root


So, let be that root




Thus for clearly , there exist such that f(x) = y


Therefore f is onto


Thus, It is a Bijective function



Question 23.

Classify the following functions as injection, surjection or bijection:

f : R → R, defined by f(x) = 3 – 4x


Answer:

TIP:One – One Function: – A function is said to be a one – one functions or an injection if different elements of A have different images in B.


So, is One – One function


⇔ a≠b


⇒ f(a)≠f(b) for all


⇔ f(a) = f(b)


⇒ a = b for all


Onto Function: – A function is said to be a onto function or surjection if every element of A i.e, if f(A) = B or range of f is the co – domain of f.


So, is Surjection iff for each , there exists such that f(a) = b


Bijection Function: – A function is said to be a bijection function if it is one – one as well as onto function.


Now, f : R → R given by f(x) = 3 – 4x


Check for Injectivity:


Let x,y be elements belongs to R i.e such that


⇒ f(x) = f(y)


⇒ 3 – 4x = 3 – 4y


⇒ x = y


Hence, f is One – One function


Check for Surjectivity:


Let y be element belongs to R i.e be arbitrary, then


⇒ f(x) = y


⇒ 3 – 4x = y



Above value of x belongs to R


Therefore for each element in R (co – domain), there exists an element in domain R.


Hence, f is onto function


Thus, Bijective function



Question 24.

Classify the following functions as injection, surjection or bijection:

f : R → R, defined by f(x) = 1 + x2


Answer:

TIP:One – One Function: – A function is said to be a one – one functions or an injection if different elements of A have different images in B.


So, is One – One function


⇔ a≠b


⇒ f(a)≠f(b) for all


⇔ f(a) = f(b)


⇒ a = b for all


Onto Function: – A function is said to be a onto function or surjection if every element of A i.e, if f(A) = B or range of f is the co – domain of f.


So, is Surjection iff for each , there exists such that f(a) = b


Bijection Function: – A function is said to be a bijection function if it is one – one as well as onto function.


Now, given by f(x) = 1 + x2


Check for Injectivity:


Let x,y be elements belongs to R i.e such that


So, from definition


⇒ f(x) = f(y)


⇒ x2 + 1 = y2 + 1


⇒ x2 = y2


⇒ ±x = ±y


Therefore, either x = y or x = – y or x ≠ y


Hence f is not One – One function


Check for Surjectivity:


1 be element belongs to R i.e be arbitrary, then


⇒ f(x) = 1


⇒ x2 + x = 1


⇒ x2 + x – 1 = 0



Above value of x not belongs to R for y < 1


Therefore f is not onto


Thus, It is also not Bijective function



Question 25.

Classify the following functions as injection, surjection or bijection:

f: R → R, defined by


Answer:

TIP:One – One Function: – A function is said to be a one – one functions or an injection if different elements of A have different images in B.


So, is One – One function


⇔ a≠b


⇒ f(a)≠f(b) for all


⇔ f(a) = f(b)


⇒ a = b for all


Onto Function: – A function is said to be a onto function or surjection if every element of A i.e, if f(A) = B or range of f is the co – domain of f.


So, is Surjection iff for each , there exists such that f(a) = b


Bijection Function: – A function is said to be a bijection function if it is one – one as well as onto function.


Now, f: R → R given by


Check for Injectivity:


Let x,y be elements belongs to R i.e. such that


⇒ f(x) = f(y)



⇒ xy2 + x = yx2 + y


⇒ xy2 + x – yx2 – y = 0


⇒ xy (y – x) + (x – y) = 0


⇒ (x – y)(1 – xy) = 0


Case i :


⇒ x – y = 0


⇒ x = y


f is One – One function


Case ii :


⇒ 1 – xy = 0


⇒ xy = 1


Thus from case i and case ii f is One – One function


Check for Surjectivity:


Let y be element belongs to R i.e be arbitrary, then


⇒ f(x) = y



⇒ x = x2y + y


⇒ x – x2y = y


Above value of x belongs to R


Therefore for each element in R (co – domain) there exists an element in domain R.


Hence, f is onto function


Thus, Bijective function



Question 26.

If f: A → B is an injection such that range of f = {a}. Determine the number of elements in A.


Answer:

TIP:One – One Function: – A function is said to be a one – one functions or an injection if different elements of A have different images in B.


So, is One – One function


⇔ a≠b


⇒ f(a)≠f(b) for all


⇔ f(a) = f(b)


⇒ a = b for all


Here, Range {f} = {a}


Since it is injective map, different elements have different images.


Thus A has only one element



Question 27.

Show that the function f : R – {3} → R – {1} given by is a bijection.


Answer:

TIP:One – One Function: – A function is said to be a one – one functions or an injection if different elements of A have different images in B.


So, is One – One function


⇔ a≠b


⇒ f(a)≠f(b) for all


⇔ f(a) = f(b)


⇒ a = b for all


Onto Function: – A function is said to be a onto function or surjection if every element of A i.e, if f(A) = B or range of f is the co – domain of f.


So, is Surjection iff for each , there exists such that f(a) = b


Bijection Function: – A function is said to be a bijection function if it is one – one as well as onto function.


Now, f: R → R given by


To Prove: – is a bijection


Check for Injectivity:


Let x,y be elements belongs to R i.e. such that


⇒ f(x) = f(y)



⇒ (x – 2)(y – 3) = (x – 3)(y – 2)


⇒ xy – 3x – 2y + 6 = xy – 2x – 3y + 6


⇒ – 3x – 2y + 2x + 3y = 0


⇒ – x + y = 0


⇒ x = y


Hence, f is One – One function


Check for Surjectivity:


Let y be element belongs to R i.e be arbitrary, then


⇒ f(x) = y



⇒ x – 2 = xy – 3y


⇒ x – xy = 2 – 3y



is a real number for all y ≠ 1.


Also, for any y


Therefore for each element in R (co – domain), there exists an element in domain R.


Hence, f is onto function


Thus, Bijective function



Question 28.

Let A = [–1, 1], Then, discuss whether the following functions from A to itself are one – one, onto or bijective:



Answer:

TIP:One – One Function: – A function is said to be a one – one functions or an injection if different elements of A have different images in B.


So, is One – One function


⇔ a≠b


⇒ f(a)≠f(b) for all


⇔ f(a) = f(b)


⇒ a = b for all


Onto Function: – A function is said to be a onto function or surjection if every element of A i.e, if f(A) = B or range of f is the co – domain of f.


So, is Surjection iff for each , there exists such that f(a) = b


Bijection Function: – A function is said to be a bijection function if it is one – one as well as onto function.


Now, here f: A → A: A = [–1, 1] given by function is


Check for Injectivity:


Let x, y be elements belongs to A i.e. such that


⇒ f(x) = f(y)



⇒ 2x = 2y


⇒ x = y


1 belongs to A then



Not element of A co – domain


Hence, f is not One – One function


Check for Surjectivity:


Let y be element belongs to A i.e be arbitrary, then


⇒ f(x) = y



⇒ x = 2y


Now,


1 belongs to A


⇒ x = 2, which not belong to A co – domain


Hence, f is not onto function


Thus, It is not Bijective function



Question 29.

Let A = [–1, 1], Then, discuss whether the following functions from A to itself are one – one, onto or bijective:

g(x) = |x|


Answer:

TIP:One – One Function: – A function is said to be a one – one functions or an injection if different elements of A have different images in B.


So, is One – One function


⇔ a≠b


⇒ f(a)≠f(b) for all


⇔ f(a) = f(b)


⇒ a = b for all


Onto Function: – A function is said to be a onto function or surjection if every element of A i.e, if f(A) = B or range of f is the co – domain of f.


So, is Surjection iff for each , there exists such that f(a) = b


Bijection Function: – A function is said to be a bijection function if it is one – one as well as onto function.


Now, here f : A → A : A = [–1, 1] given by function is g(x) = |x|


Check for Injectivity:


Let x, y be elements belongs to A i.e such that


⇒ g(x) = g(y)


⇒ |x| = |y|


⇒ x = y


1 belongs to A then


⇒ g(1) = 1 = g( – 1)


Since, it has many element of A co – domain


Hence, g is not One – One function


Check for Surjectivity:


Let y be element belongs to A i.e be arbitrary, then


⇒ f(x) = y



⇒ x = 2y


Now,


1 belongs to A


⇒ x = 2, which not belong to A co – domain


Since g attain only positive values, for negative – 1 in A (co – domain) there is no pre–image in domain A.


Hence, g is not onto function


Thus, It is not Bijective function



Question 30.

Let A = [–1, 1], Then, discuss whether the following functions from A to itself are one – one, onto or bijective:

h(x) = x2


Answer:

TIP:One – One Function: – A function is said to be a one – one functions or an injection if different elements of A have different images in B.


So, is One – One function


⇔ a≠b


⇒ f(a)≠f(b) for all


⇔ f(a) = f(b)


⇒ a = b for all


Onto Function: – A function is said to be a onto function or surjection if every element of A i.e, if f(A) = B or range of f is the co – domain of f.


So, is Surjection iff for each , there exists such that f(a) = b


Bijection Function: – A function is said to be a bijection function if it is one – one as well as onto function.


Now, here f : A → A : A = [–1, 1] given by function is h(x) = x2


Check for Injectivity:


Let x, y be elements belongs to A i.e. such that


⇒ h(x) = h(y)


⇒ x2 = y2


⇒ ±x = ±y


Since it has many elements of A co – domain


Hence, h is not One – One function


Check for Surjectivity:


Let y be element belongs to A i.e. be arbitrary, then


⇒ h(x) = y


⇒ x2 = y


⇒ x = ±√y


Since h have no pre–image in domain A.


Hence, h is not onto function


Thus, It is not Bijective function



Question 31.

Are the following set of ordered pairs functions? If so, examine whether the mapping is injective or surjective:

{(x, y): x is a person, y is the mother of x}


Answer:

TIP:One – One Function: – A function is said to be a one – one functions or an injection if different elements of A have different images in B.


So, is One – One function


⇔ a≠b


⇒ f(a)≠f(b) for all


⇔ f(a) = f(b)


⇒ a = b for all


Onto Function: – A function is said to be a onto function or surjection if every element of A i.e, if f(A) = B or range of f is the co – domain of f.


So, is Surjection iff for each , there exists such that f(a) = b


Here, It is given (x, y): x is a person, y is the mother of x


As we know each person “x” has only one biological mother


Thus,


Given relation is a function


Since more than one person may have the same mother


Function, not One – One (injective) but Onto (Surjective)



Question 32.

Are the following set of ordered pairs functions? If so, examine whether the mapping is injective or surjective:

{(a, b) : a is a person, b is an ancestor of a}


Answer:

TIP:One – One Function: – A function is said to be a one – one functions or an injection if different elements of A have different images in B.


So, is One – One function


⇔ a≠b


⇒ f(a)≠f(b) for all


⇔ f(a) = f(b)


⇒ a = b for all


Onto Function: – A function is said to be a onto function or surjection if every element of A i.e, if f(A) = B or range of f is the co – domain of f.


So, is Surjection iff for each , there exists such that f(a) = b


Here, It is given (a, b): a is a person, b is an ancestor of a


As we know any person “a” has more than one ancestor


Thus,


Given relation is not a function



Question 33.

Let A = {1, 2, 3}. Write all one – one from A to itself.


Answer:

TIP:One – One Function: – A function is said to be a one – one functions or an injection if different elements of A have different images in B.


So, is One – One function


⇔ a≠b


⇒ f(a)≠f(b) for all


⇔ f(a) = f(b)


⇒ a = b for all


We have A = {1, 2, 3}


So all one – one functions from A = {1, 2, 3} to itself are obtained by re – arranging elements of A.


Thus all possible one – one functions are:


f(1) = 1, f(2) = 2, f(3) = 3


f(1) = 2, f(2) = 3, f(3) = 1


f(1) = 3, f(2) = 1, f(3) = 2


f(1) = 1, f(2) = 3, f(3) = 2


f(1) = 3, f(2) = 2, f(3) = 1


f(1) = 2, f(2) = 1, f(3) = 3



Question 34.

If f : R → R be the function defined by f(x) = 4x3 + 7, show that f is a bijection.


Answer:

TIP:One – One Function: – A function is said to be a one – one functions or an injection if different elements of A have different images in B.


So, is One – One function


⇔ a≠b


⇒ f(a)≠f(b) for all


⇔ f(a) = f(b)


⇒ a = b for all


Onto Function: – A function is said to be a onto function or surjection if every element of A i.e, if f(A) = B or range of f is the co – domain of f.


So, is Surjection iff for each , there exists such that f(a) = b


Bijection Function: – A function is said to be a bijection function if it is one – one as well as onto function.


Now, f : R → R, defined by f(x) = 4x3 + 7


To Prove : – f : R → R is bijective defined by f(x) = 4x3 + 7


Check for Injectivity:


Let x,y be elements belongs to R i.e such that


⇒ f(x) = f(y)


⇒ 4x3 + 7 = 4y3 + 7


⇒ x3 = y3


⇒ x = y


Hence, f is One – One function


Check for Surjectivity:


Let y be element belongs to R i.e be arbitrary, then


⇒ f(x) = y


⇒ 4x3 + 7 = y


⇒ 4x3 + 7 – y = 0


Now, we know that for 3 degree equation has a real root


So, let be that root




Thus for clearly , there exist such that f(x) = y


Therefore f is onto


Thus, It is Bijective function


Hence Proved



Question 35.

Show that the exponential function f: R → R, given by f(x) = ex, is one – one but not onto. What happens if the co – domain is replaced by R0+ (set of all positive real numbers).


Answer:

TIP:One – One Function: – A function is said to be a one – one functions or an injection if different elements of A have different images in B.


So, is One – One function


⇔ a≠b


⇒ f(a)≠f(b) for all


⇔ f(a) = f(b)


⇒ a = b for all


Onto Function: – A function is said to be a onto function or surjection if every element of A i.e, if f(A) = B or range of f is the co – domain of f.


So, is Surjection iff for each, there exists such that f(a) = b


Now, given by f(x) = ex


Check for Injectivity:


Let x,y be elements belongs to R i.e such that


So, from definition


⇒ f(x) = f(y)


⇒ ex = ey



⇒ ex – y = 1


⇒ ex – y = e0


⇒ x – y = 0


⇒ x = y


Hence f is One – One function


Check for Surjectivity:


Here range of f = (0,∞) ≠ R


Therefore f is not onto


Now if co – domain is replaced by R0+ (set of all positive real numbers) i.e (0,∞) then f becomes an onto function.



Question 36.

Show that the logarithmic function f : R+0→ R given by f(x) = loga x, a > 0 is a bijection.


Answer:

TIP:One – One Function: – A function is said to be a one – one functions or an injection if different elements of A have different images in B.


So, is One – One function


⇔ a≠b


⇒ f(a)≠f(b) for all


⇔ f(a) = f(b)


⇒ a = b for all


Onto Function: – A function is said to be a onto function or surjection if every element of A i.e, if f(A) = B or range of f is the co – domain of f.


So, is Surjection iff for each , there exists such that f(a) = b


Bijection Function: – A function is said to be a bijection function if it is one – one as well as onto function.


To Prove : – Logarithmic function f : R + + → R given by f(x) = loga x, a > 0 is a bijection.


Now, f : R0+ → R given by f(x) = loga x, a > 0


Check for Injectivity:


Let x,y be elements belongs to R0+ i.e such that


So, from definition


⇒ f(x) = f(y)


⇒ loga x = loga y


⇒ loga x – loga y = 0




⇒ x = y


Hence f is One – One function


Check for Surjectivity:


Let y be element belongs to R i.e. be arbitrary, then


⇒ f(x) = y


⇒ loga x = y


⇒ x = ay


Above value of x belongs to R0+


Therefore, for all there exist x = ay such that f(x) = y .


Hence, f is Onto function.


Thus, it is Bijective also



Question 37.

If A = {1, 2, 3}, show that a one – one function f : A → A must be onto.


Answer:

TIP:One – One Function: – A function is said to be a one – one functions or an injection if different elements of A have different images in B.


So, is One – One function


⇔ a≠b


⇒ f(a)≠f(b) for all


⇔ f(a) = f(b)


⇒ a = b for all


Onto Function: – A function is said to be a onto function or surjection if every element of A i.e, if f(A) = B or range of f is the co – domain of f.


So, is Surjection iff for each , there exists such that f(a) = b


Now, f: A → A where A = {1, 2, 3} and its a One – One function


To Prove: – A is Onto function


Since it is given that f is a One – One function,


Three elements of A = {1, 2, 3} must be taken to 3 different elements of co – domain A = {1, 2, 3} under f.


Thus by definition of Onto Function


f has to be Onto function.


Hence Proved



Question 38.

If A = {1, 2, 3}, show that an onto function f : A → A must be one – one.


Answer:

TIP:One – One Function: – A function is said to be a one – one functions or an injection if different elements of A have different images in B.


So, is One – One function


⇔ a≠b


⇒ f(a)≠f(b) for all


⇔ f(a) = f(b)


⇒ a = b for all


Onto Function: – A function is said to be a onto function or surjection if every element of A i.e, if f(A) = B or range of f is the co – domain of f.


So, is Surjection iff for each , there exists such that f(a) = b


Now, f : A → A where A = {1, 2, 3} and its an Onto function


To Prove: – A is a One – One function


Let's assume f is not Onto function,


Then,


There must be two elements let it be 1 and 2 in Domain A = {1, 2, 3} whose images in co–domain A = {1, 2, 3} is same.


Also, Image of 3 under f can be only one element.


Therefore,


Range set can have at most two elements in co – domain A = {1, 2, 3}


⇒ f is not an onto function


Hence it contradicts


⇒ f must be One – One function


Hence Proved



Question 39.

Find the number of all onto functions from the set A = {1, 2, 3, …., n} to itself.


Answer:

TIP:


Onto Function: – A function is said to be a onto function or surjection if every element of A i.e, if f(A) = B or range of f is the co – domain of f.


So, is Surjection iff for each , there exists such that f(a) = b


Now, f : A → A where A = {1, 2, 3,….,n}


All onto function


It’s a permutation of n symbols 1,2,3,….n


Thus,


Total number of Onto maps from A = {1, 2, 3, …., n} to itself =


Total number of permutations of n symbols 1,2,3,….n.



Question 40.

Give examples of two one – one functions f1 and f2 from R to R such that f1 + f2 : R → R, defined by (f1 + f2)(x) = f1(x) + f2(x) is not one – one.


Answer:

TIP:One – One Function: – A function is said to be a one – one functions or an injection if different elements of A have different images in B.


So, is One – One function


⇔ a≠b


⇒ f(a)≠f(b) for all


⇔ f(a) = f(b)


a = b for all


Let, f1: R → R and f2: R → R be two functions given by (Examples)


f1(x) = x


f1(x) = – x


From above function it is clear that both are One – One functions


Now,


⇒ (f1 + f2)(x) = f1(x) + f2(x)


⇒ (f1 + f2)(x) = x – x


⇒ (f1 + f2)(x) = 0


Therefore,


f1 + f2 : R → R is a function given by


(f1 + f2)(x) = 0


Since f1 + f2 is a constant function,


Hence it is not an One – One function.



Question 41.

Give examples of two surjective function f1 and f2 from Z to Z such that f1 + f2 is not surjective.


Answer:

TIP:


Onto Function: – A function is said to be a onto function or surjection if every element of A i.e, if f(A) = B or range of f is the co – domain of f.


So, is Surjection iff for each , there exists such that f(a) = b


Let, f1: Z → Z and f2: Z → Z be two functions given by (Examples)


f1(x) = x


f1(x) = – x


From above function it is clear that both are Onto or Surjective functions


Now,


f1 + f2 : Z → Z


⇒ (f1 + f2)(x) = f1(x) + f2(x)


⇒ (f1 + f2)(x) = x – x


⇒ (f1 + f2)(x) = 0


Therefore,


f1 + f2 : Z → Z is a function given by


(f1 + f2)(x) = 0


Since f1 + f2 is a constant function,


Hence it is not an Onto/Surjective function.



Question 42.

Show that if f1 and f2 are one – one maps from R to R, then the product f1 × f2 : R → R defined by (f1 × f2)(x) = f1(x)f2(x) need not be one – one.


Answer:

TIP:One – One Function: – A function is said to be a one – one functions or an injection if different elements of A have different images in B.


So, is One – One function


⇔ a≠b


⇒ f(a)≠f(b) for all


⇔ f(a) = f(b)


a = b for all


Let, f1: R → R and f2: R → R are two functions given by


f1(x) = x


f2(x) = x


From above function it is clear that both are One – One functions


Now, f1×f2 : R → Rgiven by


⇒ (f1×f2 )(x) = f1(x)×f2(x) = x2


⇒ (f1×f2 )(x) = x2


Also,


f(1) = 1 = f( – 1)


Therefore,


f is not One – One


⇒ f1×f2 : R → R is not One – One function.


Hence Proved



Question 43.

Suppose f1 and f2 are non – zero one – one functions from R to R. Is necessarily one – one? Justify your answer. Here, is given by for all x ∈ R.


Answer:

TIP:One – One Function: – A function is said to be a one – one functions or an injection if different elements of A have different images in B.


So, is One – One function


⇔ a≠b


⇒ f(a)≠f(b) for all


⇔ f(a) = f(b)


a = b for all


Let, f1: R → R and f2: R → R are two non – zero functions given by


f1(x) = x3


f1(x) = x


From above function it is clear that both are One – One functions


Now, given by




Again,


defined by


f(x) = x2


Now,


⇒ f(1) = 1 = f( – 1)


Therefore,


f is not One – One


is not One – One function.


Hence it is not necessarily to be one – one function.



Question 44.

Given A = {2, 3, 4}, B = {2, 5, 6, 7}. Construct an example of each of the following:

an injective map from A to B


Answer:

TIP:One – One Function: – A function is said to be a one – one functions or an injection if different elements of A have different images in B.


So, is One – One function


⇔ a≠b


⇒ f(a)≠f(b) for all


⇔ f(a) = f(b)


⇒ a = b for all


Now, f : A → B, denotes a mapping such that


⇒ f = {(x,y):y = x + 3}


It can be written as follows in roster form


f = {(2,5),(3,6),(4,7)}


Hence this is injective mapping



Question 45.

Given A = {2, 3, 4}, B = {2, 5, 6, 7}. Construct an example of each of the following:

a mapping from A to B which is not injective


Answer:

TIP:One – One Function: – A function is said to be a one – one functions or an injection if different elements of A have different images in B.


So, is One – One function


⇔ a≠b


⇒ f(a)≠f(b) for all


⇔ f(a) = f(b)


⇒ a = b for all


Now, f : A → B, denotes a mapping such that


f = {(2,2),(3,5),(4,5)}


Hence this is not injective mapping



Question 46.

Given A = {2, 3, 4}, B = {2, 5, 6, 7}. Construct an example of each of the following:

a mapping from A to B.


Answer:

TIP:One – One Function: – A function is said to be a one – one functions or an injection if different elements of A have different images in B.


So, is One – One function


⇔ a≠b


⇒ f(a)≠f(b) for all


⇔ f(a) = f(b)


⇒ a = b for all


Now, f : A → B, denotes a mapping such that


f = {(2,2),(5,3),(6,4),(7,4)}


Here it is clear that every first component is from B and second component is from A


Hence this is mapping from B to A



Question 47.

Show that f : R → R, given by f(x) = x – [x], is neither one – one nor onto.


Answer:

TIP:One – One Function: – A function is said to be a one – one functions or an injection if different elements of A have different images in B.


So, is One – One function


⇔ a≠b


⇒ f(a)≠f(b) for all


⇔ f(a) = f(b)


⇒ a = b for all


Onto Function: – A function is said to be a onto function or surjection if every element of A i.e, if f(A) = B or range of f is the co – domain of f.


So, is Surjection iff for each , there exists such that f(a) = b


Now, f : A → A given by f(x) = x – [x]


To Prove: – f(x) = x – [x], is neither one – one nor onto


Check for Injectivity:


Let x be element belongs to Z i.e such that


So, from definition


⇒ f(x) = x – [x]


⇒ f(x) = 0 for


Therefore,


Range of f = [0,1] ≠ R


Hence f is not One – One function


Check for Surjectivity:


Since Range of f = [0,1] ≠ R


Hence, f is not Onto function.


Thus, it is neither One – One nor Onto function


Hence Proved



Question 48.

Let f : N → N be defined by



Show that f is a bijection.


Answer:

TIP:One – One Function: – A function is said to be a one – one functions or an injection if different elements of A have different images in B.


So, is One – One function


⇔ a≠b


⇒ f(a)≠f(b) for all


⇔ f(a) = f(b)


⇒ a = b for all


Onto Function: – A function is said to be a onto function or surjection if every element of A i.e, if f(A) = B or range of f is the co – domain of f.


So, is Surjection iff for each , there exists such that f(a) = b


Bijection Function: – A function is said to be a bijection function if it is one – one as well as onto function.


Now, suppose


f(n1) = f(n2)


If n1 is odd and n2 is even, then we have


⇒ n1 + 1 = n2 – 2


⇒ n2 – n1 = 2


Not possible


Suppose both n1 even and n2 is odd.


Then, f(n1) = f(n2)


⇒ n1 – 1 = n2 + 1


⇒ n1 – n2 = 2


Not possible


Therefore, both n1 and n2 must be either odd or even


Suppose both n1 and n2 are odd.


Then, f(n1) = f(n2)


⇒ n1 + 1 = n2 + 1


⇒ n1 = n2


Suppose both n1 and n2 are even.


Then, f(n1) = f(n2)


⇒ n1 – 1 = n2 – 1


⇒ n1 = n2


Then, f is One – One


Also, any odd number 2r + 1 in the co – domain N will have an even number as image in domain N which is


⇒ f(n) = 2r + 1


⇒ n – 1 = 2r + 1


⇒ n = 2r + 2


Any even number 2r in the co – domain N will have an odd number as image in domain N which is


⇒ f(n) = 2r


⇒ n + 1 = 2r


⇒ n = 2r – 1


Thus f is Onto function.




Exercise 2.2
Question 1.

Find gof and fog when f: R → R and g: R → R is defined by

f(x) = 2x + 3 and g(x) = x2 + 5


Answer:

Since, f:R → R and g:R → R


fog:R → R and gof:R → R


Now, f(x) = 2x + 3 and g(x) = x2 + 5


gof(x) = g(2x + 3) = (2x + 3)2 + 5


⇒ gof(x) = 4x2 + 12x + 9 + 5 = 4x2 + 12x + 14


fog (x) = f(g(x)) = f (x2 + 5) = 2 (x2 + 5) + 3


⇒ fog(x)= 2x2 + 10 + 3 = 2x2 + 13


Hence, gof(x) = 4x2 + 12x + 14 and fog (x) = 2x2 + 13



Question 2.

Find gof and fog when f: R → R and g: R → R is defined by

f(x) = 2x + x2 and g(x) = x3


Answer:

Since, f:R → R and g:R → R


fog:R → R and gof:R → R


f(x) = 2x + x2 and g(x)=x3


Now, gof(x) = g(f (x)) =g(2x + x2)


gof (x)=(2x + x2)3 = x6 + 8x3 + 6x5 + 12x4


and fog(x)=f(g(x))= f(x3)


⇒ fog(x) = 2x3 + x6


So, gof(x) = x6 + 6x5 + 12x4 + 8x3 and fog(x) = 2x3 + x6



Question 3.

Find gof and fog when f: R → R and g: R → R is defined by

f(x) = x2 + 8 and g(x) = 3x3 + 1


Answer:

Since, f:R → R and g:R → R


fog:R → R and gof:R → R


f(x)=x2 + 8 and g(x)=3x3 + 1


So, gof(x)= g(f(x))


gof(x)= g(x2 + 8)


gof(x)= 3(x2 + 8)3 + 1


⇒ gof(x)= 3(x6 + 512 + 24x4 + 192x2) + 1


⇒ gof(x)= 3x6 + 72x4 + 576x2 + 1537


Similarly, fog(x)=f(g(x))


⇒ fog(x)= f(3x3 + 1)


⇒ fog(x)=(3x3 + 1)2 + 8


⇒ fog(x)=(9x6 + 1 + 6x3) + 8


⇒ fog(x)=9x6 + 6x3 + 9


So, gof(x) = 3x6 + 72x4 + 576x2 + 1537 and fog(x) = 9x6 + 6x3 + 9



Question 4.

Find gof and fog when f: R → R and g: R → R is defined by

f(x) = x and g(x) = |x|


Answer:

Since, f:R → R and g:R → R


fog:R → R and gof:R → R


f (x) = x and g (x) = |x|


Now, gof(x)=g(f(x) =g(x)


⇒ gof(x) =|x|


and, fog(x) = f(g(x)) = f (|x|)
⇒ fog(x)=|x|


Hence, gof(x) = fog(x) = |x|



Question 5.

Find gof and fog when f: R → R and g: R → R is defined by

f(x) = x2 + 2x – 3 and g(x) = 3x – 4


Answer:

Since, f:R → R and g:R → R


fog:R → R and gof:R → R


f(x) = x2 + 2x – 3 and g(x) = 3x – 4


Now, gof(x)=g(f(x))= g(x2 + 2x – 3)


gof(x) = 3(x2 + 2x–3) – 4


⇒ gof(x)= 3x2 + 6x – 9 – 4


⇒ gof(x) = 3x2 + 6x – 13


and, fog= f(g(x)) = f(3x – 4)


fog(x) = (3x – 4)2 + 2(3x – 4) – 3


= 9x2 + 16 – 24x + 6x – 8 – 3


∴ fog(x) = 9x2 – 18x + 5


Thus, gof(x) = 3x2 + 6x – 13 and fog(x) = 9x2 – 18x + 5



Question 6.

Find gof and fog when f: R → R and g: R → R is defined by

f(x) = 8x3 and g(x) = x1/3


Answer:

Since, f:R → R and g:R → R


fog:R → R and gof:R → R


f(x) = 8x3 and


Now, gof(x) = g(f(x)) = g(8x3)



gof(x) = 2x




fog(x) = 8x


Thus, gof(x) = 2x and fog(x) = 8x



Question 7.

Let f = {(3, 1), (9, 3), (12, 4)} and g = {(1, 3), (3, 3), (4, 9), (5, 9)}. Show that gof and fog are both defined, Also, find fog and gof.


Answer:

Let f = {(3,1), (9,3), (12,4)} and


g = {(1,3), (3,3), (4,9), (5,9)}


Now,


range of f = (1, 3, 4}


domain of f = {3, 9, 12}


range of g = {3,9}


domain of g = (1, 3, 4, 5}


since, range of f ⊂ domain of g


∴ gof is well defined.


Again, the range of g ⊆ domain of f


∴ fog in well defined.


Finally, gof = {(3,3), (9 ,3), (12,9)}


fog = {(1,1) , (3,1), (4,3), (5,3)}



Question 8.

Let f = {(1, – 1), (4, – 2), (9, – 3), (16, 4)} and g = {(– 1, – 2), (– 2, – 4), (– 3, – 6), (4, 8)}. Show that gof is defined while fog is not defined. Also, find gof.


Answer:

We have,


f = {(1, – 1), (4, – 2) , (9, – 3), (16,4)} and


g = {(– 1, – 2), (– 2, – 4), (– 3, – 6), (4,8)}


Now,


Domain of f = {1,4,9,16}


Range of f = {– 1, – 2, – 3, 4}


Domain of g = (– 1, – 2, – 3,4}


Range of g = (– 2, – 4, – 6, 8}


Clearly range of f = domain of g


∴ gof is defined.


but, range of g ≠ domain of f
So, fog is not defined.


Now,


gof(1) = g(– 1)= – 2


gof(4) = g(– 2) = – 4


gof(9) = g (– 3) = – 6


gof(16) = g(4)= 8


So, gof = {(1, – 2), (4, – 4), (9 , – 6), (16,8)}



Question 9.

Let A = {a, b c}, B = {u, v, w} and let f and g be two functions from A to B and from B to A respectively defined as: f = {(a, v), (b, u), (c, w)}, g = {(u, b), (v, a), (w, c)}.

Show that f and g both are bijections and find fog and gof.


Answer:

Given, A = {a, b, c}, B = {u, v, w} and


f = A → B and g: B → A defined by


f = {(a, v), (b, u), (c, w)} and


g = {(u, b), (v, a), (w, c)}


For both f and g, different elements of domain have different


images


∴ f and g are one – one


Again, for each element in co – domain of f and g, there is a pre – image in the domain


∴ f and g are onto


Thus, f and g are bijective.


Now,


gof = {(a, a), (b, b), (c, c.)} and


fog = {(u, u), (v, v), (w, w)}



Question 10.

Find fog (2) and gof (1) when: f: R → R; f(x) = x2 + 8 and g: R → R; g(x) = 3x3 + 1.


Answer:

We have, f: R → R given by f(x) = x2 + 8 and


g : R → R given by g (x) = 3x3 + 1


fog(x) = f (g(x)) = f (3x3 + 1)


= (3x3 + 1)2 + 8


fog(2) = (3 × 8 + 1)2 + 8 = 625 + 8 = 633


Again,


gof(x) = g(f(x)) = g(x2 + 8)


= 3(x2 + 8)3 + 1


gof(1) = 3(1 + 8)3 + 1 = 2188



Question 11.

Let R+ be the set of all non – negative real numbers. If f: R+ → R+ and g: R+ → R+ are defined as f(x) = x2 and g(x) = + √x. Find fog and gof. Are they equal functions.


Answer:

We have, f : R+ → R+ given by


f (x) = x2


g: R + → R + given by



fog (x) = f(g(x)) = f(= ()2 = x


Also,


gof (x) = g(f(x)) = g(x2) = = x


Thus,


fog(x)= gof (x)


They are equal functions as their domain and range are also equal.



Question 12.

Let f: R → R and g: R → R be defined by f(x) = x2 and g(x) = x + 1. Show that fog ≠ gof.


Answer:

We have, f: R → R and g: R → R are two functions defined by


f (x)= x2 and g(x) = x + 1


Now,


fog (x) = f(g(x)) = f(x + 1) =(x + 1)2


⇒ fog(x) = x2 + 2x + 1 ……(i)


gof(x) = g(f(x)) = g(x2) = x2 + 1 ……(ii)


from (i) & (ii)


fog ≠ gof



Question 13.

Let f: R → R an g: R → R be defined by f(x) = x + 1 and g(x) = x – 1. Show that fog = gof = IR.


Answer:

Let f: R → R and g: R → R are defined as


f (x) = x + 1 and g (x) = x – 1


Now,


fog(x) = f(g(x)) = f(x – 1) = x – 1 + 1


= x =IR ……(i)


Again,


fog(x ) = f(g(x)) = g(x + 1) = x + 1 – 1


= x = IR ……(ii)


from (i)& (ii)


fog = gof = IR



Question 14.

Verify associativity for the following three mappings: f: N → Z0 (the set of non – zero integers), g: Z0→ Q and h: Q → R given by f(x) = 2x, g(x) = 1/x and h(x) = ex.


Answer:

We have, f: N → Zo, g: Z0 → Q and h: Q → R


Also, f(x) = 2x, and h(x) = ex


Now, f: N → Zo and hog: Z0 → R


∴ (hog)of: N → R


Also, gof: N → Q and h: Q → R


∴ ho(gof): N → R


Thus, (hog)of and ho(gof) exist and are function from N to set R.


Finally. (hog)of(x) = (hog)(f(x)) = (hog)(2x)



Now, ho(gof)(x) = ho(g(2x)) = h



Hence, associativity verified.



Question 15.

Consider f: N → N, g: N → N and h: N → R defined as f(x) = 2x, g(y) = 3y + 4 and h(z) = sin z for all x, y, z ∈ N. Show that ho (gof) = (hog) of.


Answer:

We have,


ho(gof)(x)=h(gof(x))=h(g(f(x)))


= h(g(2x)) = h(3(2x) + 4)


= h(6x + 4) = sin(6x + 4) ∀x ∈N


((hog)of)(x) = (hog)(f(x))= (hog)(2x)


=h(g(2x))=h(3(2x) + 4)


=h(6x + 4) = sin(6x + 4) ∀x ∈N


This shows, ho(gof) = (hog)of



Question 16.

Give examples of two functions f: N → N and g: N → N such that gof is onto, but f is not onto.


Answer:

Define f:N → N by, f(x) = x + 1 And, g: N → N by,



We first show that f is not onto.


For this, consider element 1 in co – domain N. It is clear that this element is not an image of any of the elements in domain N.


Therefore, f is not onto.



Question 17.

Give examples of two functions f: N → Z and g: Z → Z such that gof is injective, but g is not injective.


Answer:

Define f: N → Z as f(x) = x and g: N → N as g(x)=|x|.


We first show that g is not injective.


It can be observed that:


g(– 1)=| – 1| = 1


g(1) =|1| = 1


Therefore, g(– 1) = g(1), but —1 ≠ 1.


Therefore, g is not injective.


Now, gof: N → Z is defined as gof(x) = g(f(x)) =g(x)=|x|.


Let x, y ∈N such that gof(x) = gof(y).


⇒ |x|=|y|


Since x and y ∈N both are positive.


∴ |x|=|y| ⇒ x=y


Hence, gof is injective



Question 18.

If f: A → B and g: B → C are one – one functions show that gof is a one – one function.


Answer:

We have, f : A → B and g : B → C are one – one functions.


Now we have to prove : gof: A → C in one – one


let x, y ∈ A such that


gof(x) = gof(y)


g(f(x))=g(f(y))


f (x) = f (y) [As, g in one – one]


x = y [As, f in one – one]


gof is one – one function



Question 19.

If f: A → B and g: B → C are onto functions show that gof is an onto function.


Answer:

We have, f: A → B and g: B → C are onto functions.


Now, we need to prove: gof: A → C is onto.


let y ∈ C, then


gof (x) = y


g(f(x)) = y ……(i)


Since g is onto, for each element in C, there exists a preimage in B.


g(x)=y ……(ii)


From (i) & (ii)


f(x)=x


Since f is onto, for each element in B there exists a preim age in el


f(x)=x ……(iii)


From (ii)and(iii) we can conclude that for each y ∈ C, there exists a preimage in A such that gof(x) = y


∴ gof is onto.




Exercise 2.3
Question 1.

Find fog and gof, if

f(x) = ex, g(x) = logex


Answer:

f(x) = ex and g (x) = loge x


Now, fog(x) = f(g(x)) = f(loge x) = elogex= x


⇒ fog(x) = x


gof(x) = g(f(x)) = g(ex) = loge ex = x


⇒ gof(x)=x


Hence, fog(x) = x and gof(x) = x



Question 2.

Find fog and gof, if

f(x) = x2, g(x) = cos x


Answer:

f(x)= x, g (x) = cos x


Domain of f and Domain of g = R


Range of f = (0, ∞)


Range of g = (– 1,1)


∴ Range of f ⊂ domain of g ⇒ gof exist


Also, Range of g ⊂ domain of f ⇒ fog exist


Now,


gof (x) = g (f (x)) = g (x2) = cos x2


And


fog (x) = f(g(x)) = f (cos x) = cos2 x


Hence, fog(x) = cos x2 and gof(x) = cos2 x



Question 3.

Find fog and gof, if

f(x) = |x|, g(x) = sin x


Answer:

f(x)= |x| and g (x)= sin x


Range of f = (0, ∞) ⊂ Domain g (R) ⇒ gof exist


Range of g= [ – 1,1] ⊂ Domain f (R) ⇒ fog exist


Now, fog (x)= f(g(x)) = f(sin x) = |sin x| and


gof(x) = g(f(x)) = g(lxl) =sin |x|


Hence, fog(x) = |sin x| and gof(x) = sin |x|



Question 4.

Find fog and gof, if

f(x) = x + 1, g(x) = ex


Answer:

f(x) = x + 1 and g(x) = ex


Range of f = R ⊂ Domain of g= R ⇒ gof exist


Range of g = (0, ∞) = Domain of f = R ⇒ fog exist


Now,


gof(x) = g(f(x)) = g(x + 1) = ex+ 1


And


fog (x) = f(g(x))= f(ex) = ex + 1


Hence, fog(x) = ex+ 1 and gof(x) = ex + 1



Question 5.

Find fog and gof, if

f(x) = sin–1 x, g(x) = x2


Answer:

f(x) = sin – 1 x and g (x) = x2


Range of f= Domain of g = R ⇒ gof exist


Range of g= (0,∞) ⊂ Domain of f = R ⇒ fog exist


Now,


fog (x) = f(g(x)) = f(x2) = sin – 1x2 and


gof(x) = g(f(x)) = g (sin – 1 x) = (sin – 1x)2


Hence, fog(x) = sin – 1x2 and gof(x) = (sin – 1x)2



Question 6.

Find fog and gof, if

f(x) = x + 1, g(x) = sin x


Answer:

f(x) = x + 1 and g(x) = sin x


Range of f = R ⊂ Domain of g = R ⇒ gof exists


Range of g= [ – 1,1] ⊂ Domain of f ⇒ fog exists


Now,


fog(x) = f(g(x)) = f(sin x) = sin x + 1


And


gof(x) = g(f(x)) = g(x + 1) = sin(x + 1)


Hence, fog(x) = sin x + 1 and gof(x) = sin(x + 1)



Question 7.

Find fog and gof, if

f(x) = x + 1, g(x) = 2x + 3


Answer:

f(x) = x + 1 and g(x) = 2x + 3


Range of f = R ⊂ Domain of g = R ⇒ gof exists


Range of g= R ⊂ Domain of f ⇒ fog exists


Now,


fog(x) = f(g(x) = f(2x + 3) = (2x + 3) + 1= 2x + 4 and


gof(x)= g(f(x))= g(x + 1) = 2(x + 1) + 3 = 2x + 5


So, fog(x) = 2x + 4 and gof(x) = 2x + 5



Question 8.

Find fog and gof, if

f(x) = c, c ∈ R, g(x) = sin x2


Answer:

f(x) = c, c ∈ R and


g(x) = sin x2


Range of f = R ⊂ Domain of g = R ⇒ gof exists


Range of g= [ – 1,1] ⊂ Domain of f =R ⇒ fog exists


Now,


gof(x) = g(f(x)) = g(c) = sin c2 and


fog(x) = f (g(x))= f(sin x2) =c


Thus, gof(x) = sin c2 and fog(x) = c



Question 9.

Find fog and gof, if

f(x) = x2 + 2,


Answer:

f(x) = x2 + 1 and


Range of f = (2,∞) ⊂ Domain of g = R ⇒ gof exists


Range of g= R – [ – 1] ⊂ Domain of f =R ⇒ fog exists


Now,


and




Hence,



Question 10.

Let f(x) = x2 + x + 1 and g(x) = sin x. Show that fog ≠ gof.


Answer:

We have, f (x) = x2 + x + 1 and g(x) = sin x


Now,


fog(x) = f(g(x)) = f(sin x)


⇒ fog(x) = sin2 x + sin x + 1


Again, gof(x) = g(f(x)) = g (x2 + x + 1)


⇒ gof(x) = sin(x2 + x + 1)


Clearly,


fog ≠ gof



Question 11.

If f(x) = |x|, prove that fof = f.


Answer:

We have, f(x) = |x|


We assume the domain of f = R and range of f = (0,∞)


Range of f ⊂ domain of f


∴ fof exists,


Now,


fof(x) = f(f(x)) = f(|x|) = ||x|| = f(x)


∴ fof = f


Hence proved.



Question 12.

If f(x) = 2x + 5 and g(x) = x2 + 1 be two real functions, then describe each of the following functions:

(i) fog

(ii) gof

(iii) fof

(iv) f2

Also, show that fof ≠ f2.


Answer:

f(x)= 2x + 5 and g(x)= x2 + 1


The range of f = R and range of g = [1,∞]


The range of f ⊂ Domain of g (R) and range of g ⊂ domain of f (R)


∴ both fog and gof exist.


(i) fog(x) = f(g(x)) = f (x2 + 1)


= 2(x2 + 1) + 5


⇒ fog(x)=2x2 + 7


Hence fog(x) = 2x2 + 7


(ii) gof(x) = g(f(x)) = g (2x + 5)


= (2x + 5)2 + 1


gof(x)= 4x2 + 20x + 26


Hence gof(x) = 4x2 + 20x + 26


(iii) fof(x) = f(f(x)) = f(2x + 5)


= 2 (2x + 5) + 5


fof(x) = 4x + 15


Hence fof(x) = 4x + 15


(iv) f2(x) = [f(x)]2= (2x + 5)2


= 4x2 + 20x + 25


∴ from (iii) and (iv)


fof ≠ f2



Question 13.

If f(x) = sin x and g(x) = 2x be two real functions, then describe gof and fog. Are these equal functions?


Answer:

We have, f (x) = sin x and g (x) = 2x.


Domain of f and g is R


Range of f = [ – 1,1], Range of g = R


∴ Range of f ⊂ Domain g and Range of g ⊂ Domain f


fog and gof both exist.


gof(x) = g(f(x)) = g(sin x)


⇒ gof(x) = 2sin x


fog(x) = f(g(x)) = f(2x) = sin 2x


∴ gof ≠ fog



Question 14.

Let f, g, h be real functions given by f(x) = sin x, g(x) = 2x and h(x) = cos x. Prove that fog = go(fh).


Answer:

f, g and h are real functions given by f(x) = sin x, g(x) = 2x and


h(x) = cos x


To prove: fog=go(fh)


L.H.S


fog(x) = f(g(x))


= f(2x) = sin 2x


⇒ fog(x)=2sin x cos x ……(A)


R.H .S


go(fh)(x) = go(f(x).h(x))


= g(sin x cos x) = 2sin x cos x


go(fh)(x) = 2 sin x cos x ……(B)


from A and B


fog(x) = go(fh)(x)


Hence proved



Question 15.

Let f be any real function and let g be a function given by g(x) = 2x. Prove that gof = f + f.


Answer:

We are given that f is a real function and g is a function given by


g(x) = 2x


To prove; gof=f + f.


L.H.S


gof(x) = g(f(x)) = 2f(x)


=f + f = R.H.S


gof=f + f


Hence proved



Question 16.

If and g(x) = logex are two real functions, then describe functions fog and gof.


Answer:

f(x) = , g(x) = log e x


Domain of f and g are R.


Range of f= (– ∞, 1) Range of g = (0, e)


Range of f ⊂ Domain of g ⇒ gof exists


Range of g ⊂ Domain f ⇒ fog exists


∴ gof (x) = g(f(x)) = g


∴ gof (x) =


Again


fog (x) = f(g(x)) = f(log e x)


fog (x) =



Question 17.

If f: → R and g: [–1, 1] → R be defined as f(x) = tan x and respectively. Describe fog and gof.


Answer:

f: and g: [ – 1,1]R defined as f(x) = tan x and g(x) =


Range of f: let y = f(x)


⇒ y = tan x


⇒ x = tan – 1 y


Since, x ϵ , y ϵ (– ∞, ∞)


As Range of f ⊂ Domain of g


∴ gof exists.


Similarly, let y = g(x)


⇒ y =


⇒ x =


∴ Range of g is [ – 1,1]


As, Range of g ⊂ Domain of f


Hence, fog also exists


Now,


fog(x) = f(g(x)) = f


⇒ fog(x) = tan


Again,


gof(x) = g(f(x)) = g(tan x)


⇒ gof(x) =



Question 18.

If and g(x) = x2 + 1 be two real functions, then find fog and gof.


Answer:

f(x) = , g(x) = x2 + 1


Now,


Domain of f = [ – 3, ∞], domain of g = (– ∞, ∞)


Range of f = [0, ∞), range of g = [1, ∞)


Then, range of f ⊂ Domain of g and range of g ⊂ Domain of f


Hence, fog and gof exists


Now,


fog(x) = f(g(x)) = f(x2 + 1)


⇒ fog(x) =


Again,


gof(x) = g(f(x)) = g(


⇒ gof(x) =


⇒ gof(x) = x + 4



Question 19.

Let f be a real function given by . Find each of the following:

fof


Answer:

We have, f(x) =


Clearly, domain of f = [2, ∞] and range of f = [0, ∞)


We observe that range of f is not a subset of domain of f


∴ Domain of (fof) = {x: x ϵ Domain of f and f(x) ϵ Domain of f}


= {x: x ϵ [2, ∞) and ϵ [2, ∞)}


= {x: x ϵ [2, ∞) and ≥ 2}


= {x: x ϵ [2, ∞) and x – 2 ≥ 4}


= {x: x ϵ [2, ∞) and x ≥ 6}


= [6, ∞)


Now,


fof(x) = f(f(x)) = f =



Question 20.

Let f be a real function given by . Find each of the following:

fofof


Answer:

We have, f(x) =


Clearly, domain of f = [2, ∞] and range of f = [0, ∞)


We observe that range of f is not a subset of domain of f


∴ Domain of (fof) = {x: x ϵ Domain of f and f(x) ϵ Domain of f}


= {x: x ϵ [2, ∞) and ϵ [2, ∞)}


= {x: x ϵ [2, ∞) and ≥ 2}


= {x: x ϵ [2, ∞) and x – 2 ≥ 4}


= {x: x ϵ [2, ∞) and x ≥ 6}


= [6, ∞)


Clearly, range of f = [0, ∞) ⊄ Domain of (fof)


∴ Domain of ((fof)of) = {x: x ϵ Domain of f and f(x) ϵ Domain of (fof)}


= {x: x ϵ [2, ∞) and ϵ [6, ∞)}


= {x: x ϵ [2, ∞) and ≥ 6}


= {x: x ϵ [2, ∞) and x – 2 ≥ 36}


= {x: x ϵ [2, ∞) and x ≥ 38}


= [38, ∞)


Now,


(fof)(x) = f(f(x)) = f =


(fofof)(x) = (fof)(f(x)) = (fof) =


∴ fofof : [38, ∞) → R defined as


(fofof)(x) =



Question 21.

Let f be a real function given by . Find each of the following:

(fofof)(38)


Answer:

We have, f(x) =


Clearly, domain of f = [2, ∞] and range of f = [0, ∞)


We observe that range of f is not a subset of domain of f


∴ Domain of (fof) = {x: x ϵ Domain of f and f(x) ϵ Domain of f}


= {x: x ϵ [2, ∞) and ϵ [2, ∞)}


= {x: x ϵ [2, ∞) and ≥ 2}


= {x: x ϵ [2, ∞) and x – 2 ≥ 4}


= {x: x ϵ [2, ∞) and x ≥ 6}


= [6, ∞)


Clearly, range of f = [0, ∞) ⊄ Domain of (fof)


∴ Domain of ((fof)of) = {x: x ϵ Domain of f and f(x) ϵ Domain of (fof)}


= {x: x ϵ [2, ∞) and ϵ [6, ∞)}


= {x: x ϵ [2, ∞) and ≥ 6}


= {x: x ϵ [2, ∞) and x – 2 ≥ 36}


= {x: x ϵ [2, ∞) and x ≥ 38}


= [38, ∞)


Now,


(fof)(x) = f(f(x)) = f =


(fofof)(x) = (fof)(f(x)) = (fof) =


∴ fofof : [38, ∞) → R defined as


(fof)(x) = f(f(x)) = f =


(fofof)(x) = (fof)(f(x)) = (fof) =


∴ fofof : [38, ∞) → R defined as


(fofof)(x) =


(fofof)(38) =


=



Question 22.

Let f be a real function given by . Find each of the following:

f2

Also, show that fof ≠ f2.


Answer:

We have, f(x) =


Clearly, domain of f = [2, ∞] and range of f = [0, ∞)


We observe that range of f is not a subset of domain of f


∴ Domain of (fof) = {x: x ϵ Domain of f and f(x) ϵ Domain of f}


= {x: x ϵ [2, ∞) and ϵ [2, ∞)}


= {x: x ϵ [2, ∞) and ≥ 2}


= {x: x ϵ [2, ∞) and x – 2 ≥ 4}


= {x: x ϵ [2, ∞) and x ≥ 6}


= [6, ∞)


Now,


(fof)(x) = f(f(x)) = f =


∴ fof: [6, ∞) → R defined as


(fof)(x) =


f2(x) = [f(x)]2 = = x – 2


∴ f2: [2, ∞) → R defined as


f2(x) = x – 2


∴ fof ≠ f2



Question 23.

Let Find fof.


Answer:


Range of f = [0, 3] ⊂ Domain of f


∴ fof(x) = f(f(x)) = f = f


So, fof(x) =



Question 24.

If f, g: R → R be two functions defined as f(x) = |x| + x and

g(x) = |x|–x for all x ∈ R. Then, find fog and gof. Hence, find fog (–3),

fog (5) and gof(–2).


Answer:

Domain of f(x) and g(x) is R.


Range of f(x) = [0, ∞) and range of g(x) = [0, ∞)


As, range of f ⊂ Domain of g and range of g ⊂ Domain of f


So, gof and fog exists


Now,


fog(x) = f(g(x)) = f(|x|–x)


⇒ fog(x) = ||x|–x| + |x|–x


As, range of g(x) ≥ 0 so, ||x|–x| = |x|–x


So, fog(x) = ||x|–x| + |x|–x = |x|–x + |x|–x


⇒ fog(x) = 2(|x|–x)


Also,


gof(x) = g(f(x)) = g(|x| + x) = ||x| + x| – (|x| + x)


As, range of f(x) ≥ 0 so, ||x| + x| = |x| + x


So, gof(x) = ||x| + x| – (|x| + x) = |x| + x – (|x| + x) = 0


Thus, gof(x) = 0


Now, fog(– 3) = 2(| – 3|–(– 3)) = 2(3 + 3) = 6,


fog(5) = 2(|5| – 5) = 0, gof(– 2) =0




Exercise 2.4
Question 1.

State with reasons whether the following functions have inverse:

(i) f : [1, 2, 3, 4] → {10} with f = {(1, 10), (2, 10), (3, 10), (4, 10)}

(ii) g : {5, 6, 7, 8} → {1, 2, 3, 4} with g = {(5, 4), (6, 3), (7, 4), (8, 2)}

(iii) h : {2, 3, 4, 5} → {7, 9, 11, 13} with h = {(2, 7), (3, 9), (4, 11), (5, 13)}


Answer:

(i) f : [1, 2, 3, 4] → {10} with f = {(1, 10), (2, 10), (3, 10), (4, 10)}


Recall that a function is invertible only when it is both one-one and onto.


Here, we have f(1) = 10 = f(2) = f(3) = f(4)


Hence, f is not one-one.


Thus, the function f does not have an inverse.


(ii) g : {5, 6, 7, 8} → {1, 2, 3, 4} with g = {(5, 4), (6, 3), (7, 4), (8, 2)}


Recall that a function is invertible only when it is both one-one and onto.


Here, we have g(5) = 4 = g(7)


Hence, g is not one-one.


Thus, the function g does not have an inverse.


(iii) h : {2, 3, 4, 5} → {7, 9, 11, 13} with h = {(2, 7), (3, 9), (4, 11), (5, 13)}


Recall that a function is invertible only when it is both one-one and onto.


Here, observe that distinct elements of the domain {2, 3, 4, 5} are mapped to distinct elements of the co-domain {7, 9, 11, 13}.


Hence, h is one-one.


Also, each element of the range {7, 9, 11, 13} is the image of some element of {2, 3, 4, 5}.


Hence, h is also onto.


Thus, the function h has an inverse.



Question 2.

Find f-1 if it exists for f: A → B where

(i) A = {0, –1, –3, 2}; B = {–9, –3, 0, 6} & f(x) = 3x

(ii) A = {1, 3, 5, 7, 9}; B = {0, 1, 9, 25, 49, 81} & f(x) = x2


Answer:

(i) A = {0, –1, –3, 2}; B = {–9, –3, 0, 6} & f(x) = 3x


We have f : A → B and f(x) = 3x.


⇒ f = {(0, 3×0), (–1, 3×(-1)), (–3, 3×(-3)), (2, 3×2)}


∴ f = {(0, 0), (–1, –3), (–3, –9), (2, 6)}


Recall that a function is invertible only when it is both one-one and onto.


Here, observe that distinct elements of the domain {0, –1, –3, 2} are mapped to distinct elements of the co-domain {0, –3, –9, 6}.


Hence, f is one-one.


Also, each element of the range {–9, –3, 0, 6} is the image of some element of {0, –1, –3, 2}.


Hence, f is also onto.


Thus, the function f has an inverse.


We have f-1 = {(0, 0), (–3, –1), (–9, –3), (6, 2)}


(ii) A = {1, 3, 5, 7, 9}; B = {0, 1, 9, 25, 49, 81} & f(x) = x2


We have f : A → B and f(x) = x2.


⇒ f = {(1, 12), (3, 32), (5, 52), (7, 72), (9, 92)}


∴ f = {(1, 1), (3, 9), (5, 25), (7, 49), (9, 81)}


Recall that a function is invertible only when it is both one-one and onto.


Here, observe that distinct elements of the domain {1, 3, 5, 7, 9} are mapped to distinct elements of the co-domain {1, 9, 25, 49, 81}.


Hence, f is one-one.


However, the element 0 of the range {0, 1, 9, 25, 49, 81} is not the image of any element of {1, 3, 5, 7, 9}.


Hence, f is not onto.


Thus, the function f does not have an inverse.



Question 3.

Consider f : {1, 2, 3} → {a, b, c} and g : {a, b, c} → {apple, ball, cat} defined as f(1) = a, f(2) = b, f(3) = c, g(a) = apple, g(b) = ball and g(c) = cat. Show that f, g and gof are invertible. Find f-1, g-1, (gof)-1 and show that (gof)-1 = f-1og-1.


Answer:

f : {1, 2, 3} → {a, b, c} and f(1) = a, f(2) = b, f(3) = c


⇒ f = {(1, a), (2, b), (3, c)}


Recall that a function is invertible only when it is both one-one and onto.


Here, observe that distinct elements of the domain {1, 2, 3} are mapped to distinct elements of the co-domain {a, b, c}.


Hence, f is one-one.


Also, each element of the range {a, b, c} is the image of some element of {1, 2, 3}.


Hence, f is also onto.


Thus, the function f has an inverse.


We have f-1 = {(a, 1), (b, 2), (c, 3)}


g : {a, b, c} → {apple, ball, cat} and g(a) = apple, g(b) = ball, g(c) = cat


⇒ g = {(a, apple), (b, ball), (c, cat)}


Similar to the function f, g is also one-one and onto.


Thus, the function g has an inverse.


We have g-1 = {(apple, a), (ball, b), (cat, c)}


We know (gof)(x) = g(f(x))


Thus, gof : {1, 2, 3} → {apple, ball, cat} and


(gof)(1) = g(f(1)) = g(a) = apple


(gof)(2) = g(f(2)) = g(b) = ball


(gof)(3) = g(f(3)) = g(c) = cat


⇒ gof = {(1, apple), (2, ball), (3, cat)}


As the functions f and g, gof is also both one-one and onto.


Thus, the function gof has an inverse.


We have (gof)-1 = {(apple, 1), (ball, 2), (cat, 3)}


Now, let us consider f-1og-1.


We know (f-1og-1)(x) = f-1(g-1(x))


Thus, f-1og-1 : {apple, ball, cat} → {1, 2, 3} and


(f-1og-1)(apple) = f-1(g-1(apple)) = f-1(a) = 1


(f-1og-1)(ball) = f-1(g-1(ball)) = f-1(b) = 2


(f-1og-1)(cat) = f-1(g-1(cat)) = f-1(c) = 3


⇒ f-1og-1 = {(apple, 1), (ball, 2), (cat, 3)}


Therefore, we have (gof)-1 = f-1og-1.



Question 4.

Let A = {1, 2, 3, 4}; B = {3, 5, 7, 9}; C = {7, 23, 47, 79} and f : A → B, g : B → C be defined as f(x) = 2x + 1 and g(x) = x2 – 2. Express (gof)-1 and f-1og-1 as the sets of ordered pairs and verify (gof)-1 = f-1og-1.


Answer:

We have f : A → B & f(x) = 2x + 1


⇒ f = {(1, 2×1 + 1), (2, 2×2 + 1), (3, 2×3 + 1), (4, 2×4 + 1)}


∴ f = {(1, 3), (2, 5), (3, 7), (4, 9)}


Function f is clearly one-one and onto.


Thus, f-1 exists and f-1 = {(3, 1), (5, 2), (7, 3), (9, 4)}


We have g : B → C & g(x) = x2 – 2


⇒ g = {(3, 32 – 2), (5, 52 – 2), (7, 72 – 2), (9, 92 – 2)}


∴ g = {(3, 7), (5, 23), (7, 47), (9, 79)}


Function g is clearly one-one and onto.


Thus, g-1 exists and g-1 = {(7, 3), (23, 5), (47, 5), (79, 9)}


We know (gof)(x) = g(f(x))


Thus, gof : A → C and


(gof)(1) = g(f(1)) = g(3) = 7


(gof)(2) = g(f(2)) = g(5) = 23


(gof)(3) = g(f(3)) = g(7) = 47


(gof)(4) = g(f(4)) = g(9) = 79


⇒ gof = {(1, 7), (2, 23), (3, 47), (4, 79)}


Clearly, gof is also both one-one and onto.


Thus, the function gof has an inverse.


We have (gof)-1 = {(7, 1), (23, 2), (47, 3), (79, 4)}


Now, let us consider f-1og-1.


We know (f-1og-1)(x) = f-1(g-1(x))


Thus, f-1og-1 : C → A and


(f-1og-1)(7) = f-1(g-1(7)) = f-1(3) = 1


(f-1og-1)(23) = f-1(g-1(23)) = f-1(5) = 2


(f-1og-1)(47) = f-1(g-1(47)) = f-1(7) = 3


(f-1og-1)(79) = f-1(g-1(79)) = f-1(9) = 4


⇒ f-1og-1 = {(7, 1), (23, 2), (47, 3), (79, 4)}


Therefore, we have (gof)-1 = f-1og-1.



Question 5.

Show that the function f : Q → Q defined by f(x) = 3x + 5 is invertible. Also, find f-1.


Answer:

We have f : Q → Q and f(x) = 3x + 5.


Recall that a function is invertible only when it is both one-one and onto.


First, we will prove that f is one-one.


Let x1, x2ϵ Q (domain) such that f(x1) = f(x2)


⇒ 3x1 + 5 = 3x2 + 5


⇒ 3x1 = 3x2


∴ x1 = x2


So, we have f(x1) = f(x2) ⇒ x1 = x2.


Thus, function f is one-one.


Now, we will prove that f is onto.


Let y ϵ Q (co-domain) such that f(x) = y


⇒ 3x + 5 = y


⇒ 3x = y – 5



Clearly, for every y ϵ Q, there exists x ϵ Q (domain) such that f(x) = y and hence, function f is onto.


Thus, the function f has an inverse.


We have f(x) = y ⇒ x = f-1(y)


But, we found f(x) = y ⇒


Hence,


Thus, f(x) is invertible and



Question 6.

Show that the function f : R → R defined by f(x) = 4x + 3 is invertible. Find the inverse of f.


Answer:

We have f : R → R and f(x) = 4x + 3.


Recall that a function is invertible only when it is both one-one and onto.


First, we will prove that f is one-one.


Let x1, x2ϵ R (domain) such that f(x1) = f(x2)


⇒ 4x1 + 3 = 4x2 + 3


⇒ 4x1 = 4x2


∴ x1 = x2


So, we have f(x1) = f(x2) ⇒ x1 = x2.


Thus, function f is one-one.


Now, we will prove that f is onto.


Let y ϵ R (co-domain) such that f(x) = y


⇒ 4x + 3 = y


⇒ 4x = y – 3



Clearly, for every y ϵ R, there exists x ϵ R (domain) such that f(x) = y and hence, function f is onto.


Thus, the function f has an inverse.


We have f(x) = y ⇒ x = f-1(y)


But, we found f(x) = y ⇒


Hence,


Thus, f(x) is invertible and



Question 7.

Consider f : R+→ [4, ∞) given by f(x) = x2 + 4. Show that f is invertible with f-1 of f given by, where R+ is the set of all non-negative real numbers.


Answer:

We have f : R+→ [4, ∞) and f(x) = x2 + 4.


Recall that a function is invertible only when it is both one-one and onto.


First, we will prove that f is one-one.


Let x1, x2ϵ R+ (domain) such that f(x1) = f(x2)


⇒ x12 + 4 = x22 + 4


⇒ x12 = x22


∴ x1 = x2 (x1≠–x2as x1, x2ϵ R+)


So, we have f(x1) = f(x2) ⇒ x1 = x2.


Thus, function f is one-one.


Now, we will prove that f is onto.


Let y ϵ [4, ∞) (co-domain) such that f(x) = y


⇒ x2 + 4 = y


⇒ x2 = y – 4



Clearly, for every y ϵ [4, ∞), there exists x ϵ R+ (domain) such that f(x) = y and hence, function f is onto.


Thus, the function f has an inverse.


We have f(x) = y ⇒ x = f-1(y)


But, we found f(x) = y ⇒


Hence,


Thus, f(x) is invertible and



Question 8.

If, show that (fof)(x) = x for all. What is the inverse of f?


Answer:

We have


We know (fof)(x) = f(f(x))








As (fof)(x) = x = Ix (the identity function), f(x) = f-1(x).


Thus,



Question 9.

Consider f : R+→ [–5, ∞) given by f(x) = 9x2 + 6x – 5. Show that f is invertible with.


Answer:

We have f : R+→ [–5, ∞) and f(x) = 9x2 + 6x – 5.


Recall that a function is invertible only when it is both one-one and onto.


First, we will prove that f is one-one.


Let x1, x2ϵ R+ (domain) such that f(x1) = f(x2)


⇒ 9x12 + 6x1 – 5 = 9x22 + 6x2 – 5


⇒ 9x12 + 6x1 = 9x22 + 6x2


⇒ 9x12 – 9x22 + 6x1 – 6x2 = 0


⇒ 9(x12 – x22) + 6(x1 – x2) = 0


⇒ 9(x1 – x2)(x1 + x2) + 6(x1 – x2) = 0


⇒ (x1 – x2)[9(x1 + x2) + 6] = 0


⇒ x1 – x2 = 0 (as x1, x2ϵ R+)


∴ x1 = x2


So, we have f(x1) = f(x2) ⇒ x1 = x2.


Thus, function f is one-one.


Now, we will prove that f is onto.


Let y ϵ [–5, ∞) (co-domain) such that f(x) = y


⇒ 9x2 + 6x – 5 = y


Adding 6 to both sides, we get


9x2 + 6x – 5 + 6 = y + 6


⇒ 9x2 + 6x + 1 = y + 6


⇒ (3x + 1)2 = y + 6





Clearly, for every y ϵ [4, ∞), there exists x ϵ R+ (domain) such that f(x) = y and hence, function f is onto.


Thus, the function f has an inverse.


We have f(x) = y ⇒ x = f-1(y)


But, we found f(x) = y ⇒


Hence,


Thus, f(x) is invertible and



Question 10.

If f : R → R be defined by f(x) = x3 – 3, then prove that f-1 exists and find a formula for f-1. Hence, find f-1(24) and f-1(5).


Answer:

We have f : R → R and f(x) = x3 – 3.


Recall that a function is invertible only when it is both one-one and onto.


First, we will prove that f is one-one.


Let x1, x2ϵ R (domain) such that f(x1) = f(x2)


⇒ x13 – 3 = x23 – 3


⇒ x13 = x23


⇒ (x1 – x2)(x12 + x1x2 + x22) = 0


⇒ x1 – x2 = 0 (as x1, x2ϵ R+)


∴ x1 = x2


So, we have f(x1) = f(x2) ⇒ x1 = x2.


Thus, function f is one-one.


Now, we will prove that f is onto.


Let y ϵ R (co-domain) such that f(x) = y


⇒ x3 – 3 = y


⇒ x3 = y + 3



Clearly, for every y ϵ R, there exists x ϵ R (domain) such that f(x) = y and hence, function f is onto.


Thus, the function f has an inverse.


We have f(x) = y ⇒ x = f-1(y)


But, we found f(x) = y ⇒


Hence,


Thus, f(x) is invertible and


Hence, we have




Thus, f-1(24) = 3 and f-1(5) = 2.



Question 11.

A function f : R → R is defined as f(x) = x3 + 4. Is it a bijection or not? In case it is a bijection, find f-1(3).


Answer:

We have f : R → R and f(x) = x3 + 4.


Recall that a function is a bijection only if it is both one-one and onto.


First, we will check if f is one-one.


Let x1, x2ϵ R (domain) such that f(x1) = f(x2)


⇒ x13 + 4 = x23 + 4


⇒ x13 = x23


⇒ (x1 – x2)(x12 + x1x2 + x22) = 0


As x1, x2ϵ R and the second factor has no real roots,


x1 – x2 = 0


∴ x1 = x2


So, we have f(x1) = f(x2) ⇒ x1 = x2.


Thus, function f is one-one.


Now, we will check if f is onto.


Let y ϵ R (co-domain) such that f(x) = y


⇒ x3 + 4 = y


⇒ x3 = y – 4



Clearly, for every y ϵ R, there exists x ϵ R (domain) such that f(x) = y and hence, function f is onto.


Thus, the function f is a bijection and has an inverse.


We have f(x) = y ⇒ x = f-1(y)


But, we found f(x) = y ⇒


Hence,


Thus, f(x) is invertible and


Hence, we have



Thus, f-1(3) = –1.



Question 12.

If f : Q → Q, g : Q → Q are two functions defined by f(x) = 2x and g(x) = x + 2, show that f and g are bijective maps. Verify that (gof)-1 = f-1og-1.


Answer:

We have f : Q → Q and f(x) = 2x.


Recall that a function is a bijection only if it is both one-one and onto.


First, we will prove that f is one-one.


Let x1, x2ϵ Q (domain) such that f(x1) = f(x2)


⇒ 2x1 = 2x2


∴ x1 = x2


So, we have f(x1) = f(x2) ⇒ x1 = x2.


Thus, function f is one-one.


Now, we will prove that f is onto.


Let y ϵ Q (co-domain) such that f(x) = y


⇒ 2x = y



Clearly, for every y ϵ Q, there exists x ϵ Q (domain) such that f(x) = y and hence, function f is onto.


Thus, the function f is a bijection and has an inverse.


We have f(x) = y ⇒ x = f-1(y)


But, we found f(x) = y ⇒


Hence,


Thus,


Now, we have g : Q → Q and g(x) = x + 2.


First, we will prove that g is one-one.


Let x1, x2ϵ Q (domain) such that g(x1) = g(x2)


⇒ x1 + 2 = x2 + 2


∴ x1 = x2


So, we have g(x1) = g(x2) ⇒ x1 = x2.


Thus, function g is one-one.


Now, we will prove that g is onto.


Let y ϵ Q (co-domain) such that g(x) = y


⇒ x + 2 = y


∴ x = y – 2


Clearly, for every y ϵ Q, there exists x ϵ Q (domain) such that g(x) = y and hence, function g is onto.


Thus, the function g is a bijection and has an inverse.


We have g(x) = y ⇒ x = g-1(y)


But, we found g(x) = y ⇒ x = y – 2


Hence, g-1(y) = y – 2


Thus, g-1(x) = x – 2


We have (f-1og-1)(x) = f-1(g-1(x))


We found and g-1(x) = x – 2


⇒ (f-1og-1)(x) = f-1(x – 2)



We know (gof)(x) = g(f(x)) and gof : Q → Q


⇒ (gof)(x) = g(2x)


∴ (gof)(x) = 2x + 2


Clearly, gof is a bijection and has an inverse.


Let y ϵ Q (co-domain) such that (gof)(x) = y


⇒ 2x + 2 = y


⇒ 2x = y – 2



We have (gof)(x) = y ⇒ x = (gof)-1(y)


But, we found (gof)(x) = y ⇒


Hence,


Thus,


So, it is verified that (gof)-1 = f-1og-1.



Question 13.

Let A = R – {3} and B = R – {1}. Consider the function f : A → B defined by. Show that f is one-one and onto and hence find f-1.


Answer:

We have f : A → B where A = R – {3} and B = R – {1}



First, we will prove that f is one-one.


Let x1, x2ϵ A (domain) such that f(x1) = f(x2)



⇒ (x1 – 2)(x2 – 3) = (x1 – 3)(x2 – 2)


⇒ x1x2 – 3x1 – 2x2 + 6 = x1x2 – 2x1 – 3x2 + 6


⇒ –3x1 – 2x2 = –2x1 – 3x2


⇒ –3x1 + 2x1 = 2x2 – 3x2


⇒ –x1 = –x2


∴ x1 = x2


So, we have f(x1) = f(x2) ⇒ x1 = x2.


Thus, function f is one-one.


Now, we will prove that f is onto.


Let y ϵ B (co-domain) such that f(x) = y









Clearly, for every y ϵ B, there exists x ϵ A (domain) such that f(x) = y and hence, function f is onto.


Thus, the function f has an inverse.


We have f(x) = y ⇒ x = f-1(y)


But, we found f(x) = y ⇒


Hence,


Thus, f(x) is invertible and



Question 14.

Consider the function f : R+→ [–9, ∞) given by f(x) = 5x2 + 6x – 9. Prove that f is invertible with.


Answer:

We have f : R+→ [–9, ∞) and f(x) = 5x2 + 6x – 9.


Recall that a function is invertible only when it is both one-one and onto.


First, we will prove that f is one-one.


Let x1, x2ϵ R+ (domain) such that f(x1) = f(x2)


⇒ 5x12 + 6x1 – 9 = 5x22 + 6x2 – 9


⇒ 5x12 + 6x1 = 5x22 + 6x2


⇒ 5x12 – 5x22 + 6x1 – 6x2 = 0


⇒ 5(x12 – x22) + 6(x1 – x2) = 0


⇒ 5(x1 – x2)(x1 + x2) + 6(x1 – x2) = 0


⇒ (x1 – x2)[5(x1 + x2) + 6] = 0


⇒ x1 – x2 = 0 (as x1, x2ϵ R+)


∴ x1 = x2


So, we have f(x1) = f(x2) ⇒ x1 = x2.


Thus, function f is one-one.


Now, we will prove that f is onto.


Let y ϵ [–9, ∞) (co-domain) such that f(x) = y


⇒ 5x2 + 6x – 9 = y





Adding to both sides, we get









Clearly, for every y ϵ [–9, ∞), there exists x ϵ R+ (domain) such that f(x) = y and hence, function f is onto.


Thus, the function f has an inverse.


We have f(x) = y ⇒ x = f-1(y)


But, we found f(x) = y ⇒


Hence,



Question 15.

Let f : N → N be a function defined as f(x) = 9x2 + 6x – 5. Show that f : N → S, where S is the range of f, is invertible. Find the inverse of f and hence find f-1(43) and f-1(163).


Answer:

We have f : N → N and f(x) = 9x2 + 6x – 5.


We need to prove f : N → S is invertible.


Recall that a function is invertible only when it is both one-one and onto.


First, we will prove that f is one-one.


Let x1, x2ϵ N (domain) such that f(x1) = f(x2)


⇒ 9x12 + 6x1 – 5 = 9x22 + 6x2 – 5


⇒ 9x12 + 6x1 = 9x22 + 6x2


⇒ 9x12 – 9x22 + 6x1 – 6x2 = 0


⇒ 9(x12 – x22) + 6(x1 – x2) = 0


⇒ 9(x1 – x2)(x1 + x2) + 6(x1 – x2) = 0


⇒ (x1 – x2)[9(x1 + x2) + 6] = 0


⇒ x1 – x2 = 0 (as x1, x2ϵ R+)


∴ x1 = x2


So, we have f(x1) = f(x2) ⇒ x1 = x2.


Thus, function f is one-one.


Now, we will prove that f is onto.


Let y ϵ S (co-domain) such that f(x) = y


⇒ 9x2 + 6x – 5 = y


Adding 6 to both sides, we get


9x2 + 6x – 5 + 6 = y + 6


⇒ 9x2 + 6x + 1 = y + 6


⇒ (3x + 1)2 = y + 6





Clearly, for every y ϵ S, there exists x ϵ N (domain) such that f(x) = y and hence, function f is onto.


Thus, the function f has an inverse.


We have f(x) = y ⇒ x = f-1(y)


But, we found f(x) = y ⇒


Hence,


Thus, f(x) is invertible and


Hence, we have




Thus, f-1(43) = 2 and f-1(163) = 4.



Question 16.

Let f : R – → R be a function defined as. Show that f : R – → range(f) is one-one and onto. Hence, find f-1.


Answer:

We have f : R – → R and


We need to prove f : R – → range(f) is invertible.


First, we will prove that f is one-one.


Let x1, x2ϵ A (domain) such that f(x1) = f(x2)



⇒ (4x1)(3x2 + 4) = (3x1 + 4)(4x2)


⇒ 12x1x2 + 16x1 = 12x1x2 + 16x2


⇒ 16x1 = 16x2


∴ x1 = x2


So, we have f(x1) = f(x2) ⇒ x1 = x2.


Thus, function f is one-one.


Now, we will prove that f is onto.


Let y ϵ range(f) (co-domain) such that f(x) = y



⇒ 4x = 3xy + 4y


⇒ 4x – 3xy = 4y


⇒ x(4 – 3y) = 4y



Clearly, for every y ϵ range(f), there exists x ϵ A (domain) such that f(x) = y and hence, function f is onto.


Thus, the function f has an inverse.


We have f(x) = y ⇒ x = f-1(y)


But, we found f(x) = y ⇒


Hence,


Thus, f(x) is invertible and



Question 17.

If f: R → (–1, 1) defined by is invertible, find f-1.


Answer:

We have f: R → (–1, 1) and


Given that f-1 exists.


Let y ϵ (–1, 1) such that f(x) = y





⇒ 102x – 1 = y (102x + 1)


⇒ 102x – 1 = 102xy + y


⇒ 102x – 102xy = 1 + y


⇒ 102x (1 – y) = 1 + y



Taking log10 on both sides, we get






We have f(x) = y ⇒ x = f-1(y)


But, we found f(x) = y ⇒


Hence,


Thus,



Question 18.

If f: R → (0, 2) defined by is invertible, find f-1.


Answer:

We have f: R → (0, 2) and


Given that f-1 exists.


Let y ϵ (0, 2) such that f(x) = y







⇒ 2e2x = y (e2x + 1)


⇒ 2e2x = e2xy + y


⇒ 2e2x – e2xy = y


⇒ e2x (2 – y) = y



Taking ln on both sides, we get






We have f(x) = y ⇒ x = f-1(y)


But, we found f(x) = y ⇒


Hence,


Thus,



Question 19.

Let f : [–1, ∞) → [–1, ∞) is given by f(x) = (x + 1)2 – 1. Show that f is invertible. Also, find the set S = {x: f(x) = f-1(x)}


Answer:

We have f : [–1, ∞) → [–1, ∞) and f(x) = (x + 1)2 – 1


Recall that a function is invertible only when it is both one-one and onto.


First, we will prove that f is one-one.


Let x1, x2ϵ [–1, ∞) (domain) such that f(x1) = f(x2)


⇒ (x1 + 1)2 – 1 = (x2 + 1)2 – 1


⇒ (x1 + 1)2 = (x2 + 1)2


⇒ x12 + 2x1 + 1 = x22 + 2x2 + 1


⇒ x12 + 2x1 = x22 + 2x2


⇒ x12 – x22 + 2x1 – 2x2 = 0


⇒ (x12 – x22) + 2(x1 – x2) = 0


⇒ (x1 – x2)(x1 + x2) + 2(x1 – x2) = 0


⇒ (x1 – x2)[x1 + x2 + 2] = 0


⇒ x1 – x2 = 0 (as x1, x2ϵ R+)


∴ x1 = x2


So, we have f(x1) = f(x2) ⇒ x1 = x2.


Thus, function f is one-one.


Now, we will prove that f is onto.


Let y ϵ [–1, ∞) (co-domain) such that f(x) = y


⇒ (x + 1)2 – 1 = y


⇒ (x + 1)2 = y + 1




Clearly, for every y ϵ [–1, ∞), there exists x ϵ [–1, ∞) (domain) such that f(x) = y and hence, function f is onto.


Thus, the function f has an inverse.


We have f(x) = y ⇒ x = f-1(y)


But, we found f(x) = y ⇒


Hence,


Thus, f(x) is invertible and


Now, we need to find the values of x for which f(x) = f-1(x).


We have f(x) = f-1(x)




We can write



On substituting, we get


t4 = t


⇒ t4 – t = 0


⇒ t (t3 – 1) = 0


⇒ t (t – 1)(t2 + t + 1) = 0


t2 + t + 1 ≠ 0 because this equation has no real root t.


⇒ t = 0 or t – 1 = 0


⇒ t = 0 or t = 1


Case – I: t = 0



⇒ x + 1 = 0


∴ x = –1


Case – II: t = 1



⇒ x + 1 = 1


∴ x = 0


Thus, S = {0, –1}



Question 20.

Let A = {x ϵ R | –1 ≤ x ≤ 1} and let f : A → A, g : A → A be two functions defined by f(x) = x2 and g(x) = sin πx/2. Show that g-1 exists but f-1 does not exist. Also, find g-1.


Answer:

We have f : A → A where A = {x ϵ R | –1 ≤ x ≤ 1} defined by f(x) = x2.


Recall that a function is invertible only when it is both one-one and onto.


First, we will check if f is one-one.


Let x1, x2ϵ A (domain) such that f(x1) = f(x2)


⇒ x12 = x22


⇒ x12 – x22 = 0


⇒ (x1 – x2)(x1 + x2) = 0


⇒ x1 – x2 = 0 or x1 + x2 = 0


∴ x1 = ±x2


So, we have f(x1) = f(x2) ⇒ x1 = ±x2.


This means that two different elements of the domain are mapped to the same element by the function f.


For example, consider f(–1) and f(1).


We have f(–1) = (–1)2 = 1 and f(1) = 12 = 1 = f(–1)


Thus, f is not one-one and hence f-1 doesn’t exist.


Now, let us consider g : A → A defined by g(x) = sin


First, we will prove that g is one-one.


Let x1, x2ϵ A (domain) such that g(x1) = g(x2)



(in the given range)


∴ x1 = x2


So, we have g(x1) = g(x2) ⇒ x1 = x2.


Thus, function g is one-one.


Let y ϵ A (co-domain) such that g(x) = y






Clearly, for every y ϵ A, there exists x ϵ A (domain) such that g(x) = y and hence, function g is onto.


Thus, the function g has an inverse.


We have g(x) = y ⇒ x = g-1(y)


But, we found g(x) = y ⇒


Hence,


Thus, g(x) is invertible and



Question 21.

Let f be a function from R to R such that f(x) = cos (x + 2). Is f invertible? Justify your answer.


Answer:

We have f : R → R and f(x) = cos (x + 2).


Recall that a function is invertible only when it is both one-one and onto.


First, we will check if f is one-one.


Let x1, x2ϵ R (domain) such that f(x1) = f(x2)


⇒ cos (x1 + 2) = cos (x2 + 2)


As the cosine function repeats itself with a period 2π, we have


x1 + 2 = x2 + 2 or x1 + 2 = 2π + (x2 + 2)


∴ x1 = x2 or x1 = 2π + x2


So, we have f(x1) = f(x2) ⇒ x1 = x2 or 2π + x2


This means that two different elements of the domain are mapped to the same element by the function f.


For example, consider f(0) and f(2π).


We have f(0) = cos (0 + 2) = cos 2 and


f(2π) = cos (2π + 2) = cos 2 = f(0)


Thus, f is not one-one.


Hence, f is not invertible and f-1 does not exist.



Question 22.

If A = {1, 2, 3, 4} and B = {a, b, c, d}, define any four bijections from A to B. Also, give their inverse function.


Answer:

Given A = {1, 2, 3, 4} and B = {a, b, c, d}.


We need to define bijections f1, f2, f3 and f4 from A to B.


Consider f1 = {(1, a), (2, b), (3, c), (4, d)}


(1) f1 is one-one because no two elements of the domain are mapped to the same element.


f1 is also onto because each element in the co-domain has a pre-image in the domain.


Thus, f1 is a bijection from A to B.


We have f1-1 = {(a, 1), (b, 2), (c, 3), (d, 4)}


Using similar explanation, we also have the following bijections defined from A to B -


(2) f2 = {(1, b), (2, c), (3, d), (4, a)}


We have f2-1 = {(b, 1), (c, 2), (d, 3), (a, 4)}


(3) f3 = {(1, c), (2, d), (3, a), (4, b)}


We have f3-1 = {(c, 1), (d, 2), (a, 3), (b, 4)}


(4) f4 = {(1, d), (2, a), (3, b), (4, c)}


We have f4-1 = {(d, 1), (a, 2), (b, 3), (c, 4)}



Question 23.

Let A and B be two sets each with finite number of elements. Assume that there is an injective map from A to B and that there is an injective map from B to A. Prove that there is a bijection from A to B.


Answer:

Given A and B are two finite sets. There are injective maps from both A to B and B to A.


Let f be the injective map defined from A to B.


Thus, we have f is one-one.


We also know that there is a one-one mapping from B to A.


This means that each element of B is mapped to a distinct element of A.


But, B is the co-domain of f and A is the domain of f.


So, every element of the co-domain of the function f has a pre-image in the domain of the function f.


Thus, f is also onto.


Therefore, f is a bijection as it is both one-one and onto.


Hence, there exists a bijection defined from A to B.



Question 24.

If f : A → A and g : A → A are two bijections, then prove that

(i) fog is an injection

(ii) fog is a surjection


Answer:

Given f : A → A and g : A → A are two bijections. So, both f and g are one-one and onto functions.


We know (fog)(x) = f(g(x))


Thus, fog is also defined from A to A.


(i) First, we will prove that fog is an interjection.


Let x1, x2ϵ A (domain) such that (fog)(x1) = (fog)(x2)


⇒ f(g(x1)) = f(g(x2))


⇒ g(x1) = g(x2) [since f is one-one]


∴ x1 = x2 [since g is one-one]


So, we have (fog)(x1) = (fog)(x2) ⇒ x1 = x2.


Thus, function fog is an interjection.


(ii) Now, we will prove that fog is a surjection.


Let z ϵ A, the co-domain of fog.


As f is onto, we have y ϵ A (domain of f) such that f(y) = z.


However, as g is also onto and y belongs to the co-domain of g, we have x ϵ A (domain of g) such that g(x) = y.


Hence, (fog)(x) = f(g(x)) = f(y) = z.


Here, x belongs to the domain of fog (A) and z belongs to the co-domain of fog (A).


Thus, function fog is a surjection.




Very Short Answer
Question 1.

Which one of the following graphs represent a function?




Answer:

(a) It have unique image therefore a function


(b) It have more than one image



Question 2.

Which one of the following graphs represent a one-one function?




Answer:

Formula:-


(i) A function is one-one function or an injection if


f(x)=f(y)


⇒ x=y for all x, y ∈ A


or f(x)f(y)


⇒ xy for all x, y ∈ A


(a) It is not one-one function as it has same image on x axis


(b) It is one-one function s it have unique image



Question 3.

If A = {1, 2, 3} and B = {a, b}, write total number of functions from A to B.


Answer:

Formula:-if A and B are two non-empty finite sets containing m and n


(i) Number of function from A to B = nm


(ii) Number of one–one function from A to B


(iii) Number of one-one and onto function from A to B


(iv) Number of onto function from A to B=


given: -


A = {1, 2, 3} and B = {a, b}


n(A)=3, and n(B)=2


total number of functions=23=8



Question 4.

If A = {a, b, c} and B = {–2, –1, 0, 1, 2}, write total number of one-one functions from A to B.


Answer:

Formula:-


(I)A function is one-one function or an injection if


f(x)=f(y)


⇒ x=y for all x, y ∈ A


or f(x)f(y)


⇒ xy for all x, y ∈ A


(II)if A and B are two non-empty finite sets containing m and n


(i) Number of function from A to B = nm


(ii) Number of one–one function from A to B


(iii) Number of one-one and onto function from A to B


(iv) Number of onto function from A to B=


Let f: A→B be one-one function


F(a)=3 and f(B)=5


Using formula


Number of one–one function from A to B


3C5.5!=60



Question 5.

Write total number of one-one functions from set A = {1, 2, 3, 4} to set B = {a, b c}.


Answer:

Formula:-


(I) A function is one-one function or an injection if


f(x)=f(y)


⇒ x=y for all x, y ∈ A


or f(x)f(y)


⇒ xy for all x, y ∈ A


(II)if A and B are two non-empty finite sets containing m and n


(i) Number of function from A to B = nm


(ii) Number of one–one function from A to B


(iii) Number of one-one and onto function from A to B


(iv) Number of onto function from A to B=


F(A)=4 and f(B)=3


Using formula


Number of one–one function from A to B


Number of one-one function from A to B=0



Question 6.

If f: R→ R is defined by f(x) = x2, write f–1(25).


Answer:

Formula:-


(i)A function f: X → Y is defined to be invertible, if there exists a function g : Y → X


such that gof =Ix and fog = Iy. The function g is called the inverse of f and is denoted by f-1


f(x)=y


f-1(y)=x


x2=25


x=-5,5


f-1(25) ={-5,5}



Question 7.

If f : C → C is defined by f(x) = x2, write f–1(–4). Here, C denotes the set of all complex numbers.


Answer:

Formula:-


(i)A function f: X → Y is defined to be invertible, if there exists a function g : Y → X


such that gof =Ix and fog = Iy .The function g is called the inverse of f and is denoted by f-1


f(x)=y


f-1(y)=x


f(x)=-4


x2=-4


x=2i,-2i



Question 8.

If f: R → R is given by f(x) = x3, write f–1(1).


Answer:

Formula:-


(i)A function f: X → Y is defined to be invertible, if there exists a function g : Y → X


such that gof =Ix and fog = Iy. The function g is called the inverse of f and is denoted by f-1


f(x)=y


f-1(y)=x


f-1(1) =x


f(x)=1


x3=1


x3-1=0


(x-1) (x2+x+1)=0


X=1



Question 9.

Let C denote the set of all complex numbers. A function f : C → C is defined by f(x) = x3.

Write f–1(1).


Answer:

Formula:-


(i)A function f: X → Y is defined to be invertible, if there exists a function g : Y → X


such that gof =Ix and fog = Iy. The function g is called the inverse of f and is denoted by f-1


f(x)=y


f-1(y)=x


f-1(1) =x


f(x)=1


x3=1


x3-1=0


(x-1) (x2+x+1)=0


x=1, w,w2



Question 10.

Let f be a function from C (set of all complex numbers) to itself given by f(x) = x3. Write f–1(–1).


Answer:

Formula:-


(i)A function f: X → Y is defined to be invertible, if there exists a function g : Y → X


such that gof =Ix and fog = Iy .The function g is called the inverse of f and is denoted by f-1


f(x)=y


f-1(y)=x


f(x)=-1


f-1(-1)=x


x3=-1


x3+1=0


(x+1)(x2-x+1)=0


x=-1,-w,-w2



Question 11.

Let f : R → R be defined by f(x) = x4, write f–1(1).


Answer:

Formula:-


(i)A function f : X → Y is defined to be invertible, if there exists a function g : Y → X


such that gof =Ix and fog = Iy .The function g is called the inverse of f and is denoted by f-1


f(x)=y


f-1(y)=x


f(x)=1


f-1(1)=x


x4=1


x4-1=0


(x-1)(x2+1)=0


x=-1,1


f–1(1)={-1,1}



Question 12.

If f : C → C is defined by f(x) = x4, f–1(1).


Answer:

Formula:-


(i)A function f : X → Y is defined to be invertible, if there exists a function g : Y → X


such that gof =Ix and fog = Iy .The function g is called the inverse of f and is denoted by f-1


f(x)=y


f-1(y)=x


f(x)=1


f-1(1)=x


x4=1


x4-1=0


(x-1)(x2+1)=0


x=-1,1,i,-i


f–1(1)={-1,-i,1,i}



Question 13.

If f :R→R is defined by f(x) = x2, f–1(–25).


Answer:

Formula:-


(i)A function f : X → Y is defined to be invertible, if there exists a function g : Y → X


such that gof =Ix and fog = Iy .The function g is called the inverse of f and is denoted by f-1


f(x)=y


f-1(y)=x


x2=-25


but x should be Real number




Question 14.

If f : C → C is defined by f(x) = (x – 2)3, write f–1(–1).


Answer:

Formula:-


(i)A function f : X → Y is defined to be invertible, if there exists a function g : Y → X


such that gof =Ix and fog = Iy .The function g is called the inverse of f and is denoted by f-1


f(x)=y


f-1(y)=x


(x – 2)3=-1


x-2=-1,x-2=w and x-2=-w2


x=1,-w+2,2-w2


f-1(25)={ 1,2-w,2-w2}



Question 15.

If f :R→R is defined by f(x) = 10x –7, then write f–1(x).


Answer:

Formula:-


(i)A function f : X → Y is defined to be invertible, if there exists a function g : Y → X


such that gof =Ix and fog = Iy .The function g is called the inverse of f and is denoted by f-1


f-1(x)=y


f(y)=x


10y-7=x





Question 16.

Let be a function defined by f(x) = cos[x]. Write range (f).


Answer:

Given:-


(i)


(ii)f(x)=cos[x]


Domain=


For f(x)= cos [x]


Range ={1,cos1,cos2}



Question 17.

If f : R → R defined by f(x) = 3x – 4 is invertible then write f–1(x).


Answer:

Given:- (i) f : R → R


(ii) f(x)=3x-4


Formula:-


(i)A function f : X → Y is defined to be invertible, if there exists a function g : Y → X


such that gof =Ix and fog = Iy .The function g is called the inverse of f and is denoted by f-1


For f-1(x)=y


f(y)=x


3y – 4 =x





Question 18.

If f : R → R, g : R → R are given by f(x) = (x + 1)2 and g(x) = x2 + 1, then write the value of fog (–3).


Answer:

Formula:-


(I)Let f : AB and g : BC be two functions.


Then, the composition of f andg, denoted by g o f, is defined as the function g o f : AC


given by g o f (x) = g (f (x))


Given:-


(i) f : R → R


(ii) g : R → R


(iii) f(x) = (x + 1)2


(iv) g(x) = x2 + 1


fog(-3)=f(g(-3))


fog(-3)=f((-3)2+1)


fog(-3)=f(10)


fog(-3)=(10+1)2


fog(-3)=121



Question 19.

Let A = {x ϵR : –4 ≤ x ≤ x ≤ 4 and x ≠ 0} and f : A → R be defined by Write the range of f.


Answer:

Given:-


(i)


(ii) f : A → R


(iii)


For


Range ={-1,1}



Question 20.

Let be defined by f(x) = sinx. If f is a bijection, write set A.


Answer:

Formula:-


(i)A function is a bijection if it is one-one as well as onto


(ii)A function is onto function or surjection if


Range (f)=co-domain(f)


Given:-


(i)


(ii)f(x)=sinx


(ii) f is bijection


For f(x)=sinx


Codomain =range


Set A=[-1,1]



Question 21.

Let f : R → R+ be defined by f(x) = ax, a > 0 and a ≠ 1. Write f–1(x).


Answer:

Given:-


(i)f : R → R+


(ii)f(x) = ax, a > 0 and a ≠ 1


Let


f(y)=x


ay=x





Question 22.

Let f : R – {–1} → R – {1} be given by Write f–1(x).


Answer:

Given:-


(i)f : R – {–1} → R – {1}


(ii)


F(y)=x



Y=xy+x





Question 23.

Let be a function defined as


Answer:

Formula:-


(i)A function f : X → Y is defined to be invertible, if there exists a function g : Y → X


such that gof =Ix and fog = Iy .The function g is called the inverse of f and is denoted by f-1


Given:-




F(y)=x



2y-3x-5xy=0





Question 24.

Let f : R → R, g : R → R be two functions defined by f(x) = x2 + x + 1 and g(x) = 1 – x2. Write fog(–2).


Answer:

Formula :- (I)Let f : AB and g : BC be two functions.


Then, the composition of f and g, denoted by g o f, is defined as the function g o f : AC


given by g o f (x) = g (f (x))


Given:-


(i)f : R → R


(ii)g : R → R


(iii) f(x) = x2 + x + 1


(iv)g(x) = 1 – x2


Fog(-2)=f(g(-2))


Fog(-2)=f(1-(-2)2)


Fog(-2)=f(-3)


Fog(-2)=(-3)2-3+1=7



Question 25.

Let f : R →R be defined as Write fof–1(1).


Answer:

Formula:-


(i)A function f : X → Y is defined to be invertible, if there exists a function g : Y → X


such that gof =Ix and fog = Iy .The function g is called the inverse of f and is denoted by f-1


(II)Let f : AB and g : BC be two functions.


Then, the composition of f and g, denoted by g o f, is defined as the function g o f : AC


given by g o f (x) = g (f (x))


Given:-


(i)f : R →R



F(y)=x



2y-3-4x=0



Now





Question 26.

Let f be an invertible real function. Write (f–1 of) (1) + (f–1 of) (2) + … +(f–1 of) (100).


Answer:

Formula:-


(i)A function f : X → Y is defined to be invertible, if there exists a function g : Y → X


such that gof =Ix and fog = Iy .The function g is called the inverse of f and is denoted by f-1


(II)Let f : AB and g : BC be two functions.


Then, the composition of f and g, denoted by g o f, is defined as the function g o f : AC


given by g o f (x) = g (f (x))


Given:-


(i)f be an invertible real function


(f–1 of) (1) + (f–1 of) (2) + … +(f–1 of) (100)


=1+2+3+………+100




Question 27.

Let A = {1, 2, 3, 4} and B = {a, b} be two sets. Write total number of onto functions from A to B.


Answer:

Formula:-


(I)A function is onto function or surjection if


Range (f)=co-domain(f)


(II)if A and B are two non-empty finite sets containing m and n


(i) Number of function from A to B = nm


(ii) Number of one–one function from A to B


(iii) Number of one-one and onto function from A to B


(iv) Number of onto function from A to B=


Given:-


(i)A = {1, 2, 3, 4}=4


(ii)B = {a, b}=2


Using formula (iv)


Number of onto function from A to B=


Where m=4,n=2



=-2+16=14



Question 28.

Write the domain of the real function


Answer:

where x is for all real number


Then,


domain=R



Question 29.

Write the domain of the real function


Answer:


When x<0 negative





When x>0



Domain=



Question 30.

Write whether f : R → R given by is one-one, many-one, onto or into.


Answer:

(I)A function is one-one function or an injection if


f(x)=f(y)x=y for all x,yA


or f(x)f(y)xy for all x,yA


(II) A function is onto function or surjection if


Range (f)=co-domain(f)


(III) A function is not onto function, then


is always an onto function


Given:-


(i)f : R → R


(ii)



=x ± x


=0, 2x


Now putting x=0



Again putting x=-1



Hence f is many one



Question 31.

If f(x) = x + 7 and g(x) = x – 7, x ϵ R, write fog(7).


Answer:

Formula:-


(i)Let f : AB and g : BC be two functions.


Then, the composition of f and g, denoted by g o f, is defined as the function g o f : AC


given by g o f (x) = g (f (x))


Given:-


(i)f(x) = x + 7


(ii)g(x) = x – 7, x ϵ R


Fog(7)=f(g(7))


Fog(7)=f(7-7)


Fog(7)=f(0)


Fog(7)=0+7


Fog(7)=7



Question 32.

What is the range of the function


Answer:


=±1


Range of f={-1,1}



Question 33.

If f : R → R be defined by f(x) = (3 – x3)1/3, then find fof(x).


Answer:

Formula:-


(i)Let f : AB and g : BC be two functions.


Then, the composition of f and g, denoted by g o f, is defined as the function g o f : AC


given by g o f (x) = g (f (x))


Given:-


(i) f : R → R



Fof(x)=f(f(x))






Question 34.

If f : R → R is defined by f(x) = 3x + 2, find f(f (x)).


Answer:

Given:-


(i)f : R → R


F(f(x))=f(3x+2)


F(f(x))=3(3x+2)+2


F(f(x))=9x+8



Question 35.

Let A = {1, 2, 3}, B = {4, 5, 6, 7} and let f = {(1, 4), (2, 5), (3, 6)} be a function from A to B. State whether f is one-one or not.


Answer:

Given:-


(i) A = {1, 2, 3}


(ii) B = {4, 5, 6, 7}


(iii) f = {(1, 4), (2, 5), (3, 6)}


each element has a unique image


hence ,f is one-one



Question 36.

If f : {5, 6} → {2, 3} and g : {2, 3} → {5, 6} are given by f = {(5, 2), (6, 3)} and g = {(2, 5), (3, 6)}, find fog.


Answer:

Formula:-


(i)Let f : AB and g : BC be two functions.


Then, the composition of f and g, denoted by g o f, is defined as the function g o f : AC


given by g o f (x) = g (f (x))


Given:-


(i) f : {5, 6} → {2, 3}


(ii) g : {2, 3} → {5, 6}


(iv) f = {(5, 2), (6, 3)}


(v) g = {(2, 5), (3, 6)}


for fog(2)=f(g(2))


fog(2)=f(5)


fog(2)=2



Question 37.

Let f : R → R be the function defined by f(x) = 4x – 3 for all x ϵ R. Then write f–1.


Answer:

Formula:-


(i)A function f : X → Y is defined to be invertible, if there exists a function g : Y → X


such that gof =Ix and fog = Iy .The function g is called the inverse of f and is denoted by f-1


Given:-


(i) f : R → R


(ii) f(x) = 4x – 3 for all x ϵ R.


f(x)=y


4x-3=y






Question 38.

Which one the following relations on A = {1, 2, 3} is a function?

f = {(1, 3), (2, 3), (3, 2)}, g = {(1, 2), (1, 3), (3, 1)}.


Answer:

Given:-


(i) A = {1, 2, 3}


(ii)f = {(1, 3), (2, 3), (3, 2)}


(iii)g = {(1, 2), (1, 3), (3, 1)}.


In case of set A and f


Every element in A has a unique image in f


So, f is a function


In case of set A and g


Only one element has image in g


So, g is not a function



Question 39.

Write the domain of the real function f defined by


Answer:


⇒ 25-x2 ≥0


⇒ -(x+5)(x-5)≥0


⇒ (x+5)(x-5)≤0


⇒ x≤-5 or 5


Domain = [-5,5]



Question 40.

Let A = {a, b, c, d} and f : A → A be given by f = {(a, b), (b, d), (c, a), (d, c)}, write f–1.


Answer:

Formula:-


(i)A function f : X → Y is defined to be invertible, if there exists a function g : Y → X


such that gof =Ix and fog = Iy .The function g is called the inverse of f and is denoted by f-1


(ii)A function is onto function or surjection if


Range (f)=co-domain(f)


Given:-


(i)A = {a, b, c, d}


(ii)f : A → A


(iii)f = {(a, b), (b, d), (c, a), (d, c)}


f is one-one since each element of A is assigned to distinct element of the set A. Also, f is onto since f (A) = A.


f-1= {(b, a), (d, b), (a, c), (c, d)}.



Question 41.

Let f, g : R → R be defined by f(x) = 2x + 1 and g(x) = x2 – 2 for all x ϵ R, respectively. Then, find gof.


Answer:

Formula:-


(i)Let f : AB and g : BC be two functions.


Then, the composition of f and g, denoted by g o f, is defined as the function g o f : AC


given by g o f (x) = g (f (x))


Given:-


(i)f, g : R → R


(ii)f(x) = 2x + 1


(ii)g(x) = x2 – 2 for all x ϵ R


gof(x)=g(f(x))


gof(x)=g(2x+1)


gof(x)=(2x+1)2-2


gof(x)=4x2+4x-1



Question 42.

If the mapping f : {1, 3, 4} → {1, 2, 5} and g : {1, 2, 5} → {1, 3}, given by

f = {(1, 2), (3, 5), (4, 1)} and g = {(2, 3), (5, 1), (1, 3), write fog.


Answer:

Formula:-


(i)Let f : AB and g : BC be two functions.


Then, the composition of f and g, denoted by g o f, is defined as the function g o f : AC


given by g o f (x) = g (f (x))


Given:-


(i)f : {1, 3, 4} → {1, 2, 5}


(ii)g : {1, 2, 5} → {1, 3}


(iii)f = {(1, 2), (3, 5), (4, 1)}


(iv)g = {(2, 3), (5, 1), (1, 3)


fog(1)=f(g(1))=f(3)=5


fog(2)=f(g(2))=f(3)=5


fog(5)=f(g(5))=f(1)=2


fog={(1,5)(2,5)(5,2)}



Question 43.

If a function g = {(1, 1), (2, 3), (3, 5), (4, 7)} is described by g(x) = αx + β, find the values of α and β.


Answer:

Given:-


(i)g={(1, 1), (2, 3), (3, 5), (4, 7)}


(ii)g(x) = αx + β


For x=1 and αx + β


g(1)=α(1)+ β=1


⇒ α+ β=1


For x=2


g(2)=α(2)- β=3


⇒ 2α- β=3


Similarly with g(3) and g(4)


Using above value


α=2


β=1



Question 44.

If f(x) = 4 – (x – 7)3, write f–1(x).


Answer:

Formula:-


(i)A function f : X → Y is defined to be invertible, if there exists a function g : Y → X


such that gof =Ix and fog = Iy .The function g is called the inverse of f and is denoted by f-1


Given:-


(i) f(x) = 4 – (x – 7)3


Let f(x)=y


y= 4 – (x – 7)3*





Question 45.

Write the domain of the real function


Answer:

where x is not for real number


Domain=




Mcq
Question 1.

Mark the correct alternative in each of the following:

Let A = {x ϵ R : –1 ≤ x ≤ 1} = B and C = {x ϵ R : X ≥ 0} and let S = {(x, y) ϵ A × B : x2 + y2 = 1} and S0 = {(x, y) ϵ A × C : x2 + y2 = 1} Then

A. S defines a function from A to B

B. S0 defines a function from A to C

C. S0 defines a function from A to B

D. S defines a function from A to C


Answer:

Given that


A = {x ϵ R: –1 ≤ x ≤ 1} = B


C = {x ϵ R: X ≥ 0}


S = {(x, y) ϵ A × B: x2 + y2 = 1}


S0 = {(x, y) ϵ A × C: x2 + y2 = 1}


x2 + y2 = 1


⇒ y2 =1 - x2



∴ y ϵ B


Hence, S defines a function from A to B.


Question 2.

Mark the correct alternative in each of the following:

f: R → R given by is

A. injective B. surjective

C. bijective D. none of these


Answer:

Given function is f: R → R given


f(x) = x + √x2


For this function if we take x = 2,


f(x) = 2 + √4


⇒ f(x) =2


For this function if we take x = -2,


f(x) = -2 + √4


⇒ f(x) = 0


So, in general for every negative x, f(x) will be always 0. There is no x ϵ R for which f(x) ϵ (-∞, 0).


Hence, it is neither injective nor surjective and so it is not bijective either.


Question 3.

Mark the correct alternative in each of the following:

If f : A → B given by 3f(x) + 2–x = 4 is a bijection, then

A. A = {x ϵ R : –1 < x < ∞}, B = {x ϵ R : 2 < x < 4}

B. A = {x ϵ R : –3 < x < ∞}, B = {x ϵ R : 0 < x < 4}

C. A = {x ϵR : –2 < x < ∞}, B = {x ϵ R : 0 < x < 4}

D. none of these


Answer:

Given that f: A → B given by 3f(x) + 2–x = 4 is a bijection.


3f(x) + 2–x = 4


⇒ 3f(x) = 4 - 2–x


⇒ 4 - 2–x ≥ 0


⇒ 4 ≥ 2–x


⇒ 2 ≥ -x


⇒ x ≥ -2


So, x ϵ (-2, ∞)


But, for x =0, f(x) = 1.


Hence, the correct option is none of these.


Question 4.

Mark the correct alternative in each of the following:

The function f : R → R defined by f(x) = 2x + 2|x| is

A. one-one and onto

B. many-one and onto

C. one-one and into

D. many-one and into


Answer:

Given that f: R → R where f(x) = 2x + 2|x|


Here, for each value of x we will get different value of f(x).


So, it is one-one.


Also, f(x) is always positive for x ϵ R.


There is no x ϵ R for which f(x) ϵ (-∞, 0).


So, it is into.


Hence, the given function is one-one and into.


Question 5.

Mark the correct alternative in each of the following:

Let the function f : R – {–b} → R – {1} be defined by then

A. f is one-one but not onto

B. f is onto but not one-one

C. f is both one-one and onto

D. none of these


Answer:

Given that f: R – {–b} → R – {1} where



Here, f(x) = f(y) only when x=y.


Hence, it is one-one.


Now, f(x) = y



⇒ x + a = y(x + b)


⇒ x – yx = yb – a



So, x ϵ R – {1}


Hence, it is onto.


Question 6.

Mark the correct alternative in each of the following:

The function f : A → B defined by f(x) = –x2 + 6x – 8 is a bijection, if

A. A = (–∞, 5] and B = (–∞, 1]

B. A = [–3, ∞] and B = (–∞, 1]

C. A = (–∞, 3] and B = [1, ∞)

D. A = [3, ∞) and B = [1, ∞)


Answer:

Given that f: A → B defined by f(x) = –x2 + 6x – 8 is a bijection.


f(x) = –x2 + 6x – 8


⇒ f(x) = –(x2 - 6x + 8)


⇒ f(x) = - (x2 - 6x + 8 + 1 - 1)


⇒ f(x) = - (x2 - 6x + 9 - 1)


⇒ f(x) = - [(x – 3)2 – 1]


Hence, x ϵ (–∞, 5] and f(x) ϵ (–∞, 1]


Question 7.

Mark the correct alternative in each of the following:

Let A = {x ϵR : –1 ≤ x ≤ 1} = B. Then, the mapping f : A → B given by f(x) = x |x| is

A. injective but not surjective

B. surjective but not injective

C. bijective

D. none of these


Answer:

Given that A = {x ϵ R: –1 ≤ x ≤ 1} = B. Then, the mapping f: A → B given by f(x) = x |x|.


For x <0, f(x) <0


⇒ y = -x2


⇒ x = √-y, which is not possible for x > 0.


Hence, f is one-one and onto.


∴ the given function is bijective.


Question 8.

Mark the correct alternative in each of the following:

Let f: R → R be given by f(x) = [x]2 + [x + 1]–3, where [x] denotes the greatest integer less than or equal to x. Then, f(x) is

A. many-one and onto

B. many-one and into

C. one-one and into

D. one-one and onto


Answer:

Given that f: R → R be given by f(x) = [x]2 + [x + 1] – 3


As [x] is the greatest integer so for different values of x, we will get same value of f(x).


[x]2 + [x + 1] will always be an integer.


So, f is many-one.


Similarly, in this function co domain is mapped with at most one element of domain because for every x ϵ R, f(x) ϵ Z.


So, f is into.


Question 9.

Mark the correct alternative in each of the following:

Let M be the set of all 2 × 2 matrices with entries from the set R of real numbers. Then the function f : M → R defined by f(A) = |A| for every A ϵ M, is

A. one-one and onto

B. neither one-one nor onto

C. one-one not one-one

D. onto but not one-one


Answer:

Given that M is the set of all 2 × 2 matrices with entries from the set R of real numbers. Then the function f: M → R defined by f(A) = |A| for every A ϵ M.


If f(a) =f(b)


⇒ |a| = |b|


But this does not mean that a=b.


So, f is not one-one.


As a ≠ b but |a|=|b|


So, f is onto.


Question 10.

Mark the correct alternative in each of the following:

The function f : [0, ∞) → R given by f(x) is

A. one-one and onto

B. one-one but not onto

C. onto but not one-one

D. neither one-one nor onto


Answer:

Given that f: [0, ∞) → R where


Let f(x) = f(y)



⇒ xy + x = xy + y


⇒ x = y


So, f is one-one.


Now, y = f(x)



⇒ xy + y = x


⇒ y = x – xy



Here, y≠1 i.e. y ϵ R.


So, f is not onto.


Question 11.

Mark the correct alternative in each of the following:

The range of the function f(x) = 7–xPx–3 is

A. {1, 2, 3, 4, 5}

B. {1, 2, 3, 4, 5, 6}

C. {1, 2, 3, 4}

D. {1, 2, 3}


Answer:

Given that f(x) = 7–xPx–3


Here, 7-x ≥ x-3


⇒ 10 ≥ 2x


⇒ 5 ≥ x


So, domain = {3, 4, 5}


Range = {4P0, 3P1, 2P2} = {1, 3, 2}


Question 12.

Mark the correct alternative in each of the following:

A function f from the set on natural numbers to integers defined by

..is

A. neither one-one nor onto

B. one-one but not onto

C. onto but not one-one

D. one-one and onto both


Answer:

Given that a function f from the set on natural numbers to integers where



For n is odd


Let f(n) = f(m)



⇒ n = m


For n is even


Let f(n) = f(m)



⇒ n = m


So, f is one-one.


Also, each element of y is associated with at least one element of x, so f is onto.


Hence, f is one-one and onto.


Question 13.

Mark the correct alternative in each of the following:

Let f be an injective map with domain {x, y, z} and range {1, 2, 3} such that exactly one of the following statements is correct and the remaining are false.

f(x) = 1, f(y) ≠ 1, f(z) ≠ 2.

The value of f–1(1) is

A. x

B. y

C. z

D. none of these


Answer:

Given that f is an injective map with domain {x, y, z} and range {1, 2, 3}.


Case-1


Let us assume that f(x) =1 is true and f(y) ≠ 1, f(z) ≠ 2 is false.


Then f(x) = 1, f(y) = 1 and f(z) = 2.


This violates the injectivity of f because it is one-one.


Case-2


Let us assume that f(y) ≠1 is true and f(x)= 1, f(z) ≠ 2 is false.


Then f(x) ≠ 1, f(y) ≠ 1 and f(z) = 2.


This means there is no pre image of 1 which contradicts the fact that the range of f is {1, 2, 3}.


Case-3


Let us assume that f(z) ≠2 is true and f(x)= 1, f(y) ≠ 1 is false.


Then f(z) ≠ 2, f(y) = 1 and f(x) ≠1.


⇒ f–1(1) = y


Question 14.

Mark the correct alternative in each of the following:

Which of the following functions from Z to itself are bijections?

A. f(x) = x3

B. f(x) = x + 2

C. f(x) = 2x + 1

D. f(x) = x2 + x


Answer:

a. f(x) = x3


⇒ For no value of x ϵ Z, f(x) = 2.


Hence, it is not bijection.


b. f(x) = x + 2


If f(x) = f(y)


⇒ x + 2 = y + 2


⇒ x = y


So, f is one-one.


Also, y = x + 2


⇒ x = y – 2 ϵ Z


So, f is onto.


Hence, this function is bijection.


c. f(x) = 2x + 1


If f(x) = f(y)


⇒ 2x + 1 = 2y + 1


⇒ x = y


So, f is one-one.


Also, y = 2x + 1


⇒ 2x = y – 1



So, f is into because x can never be odd for any value of y.


d. f(x) = x2 + x


For this function if we take x = 2,


f(x) = 4 + 2


⇒ f(x) = 6


For this function if we take x = -2,


f(x) = 4 - 2


⇒ f(x) = 2


So, in general for every negative x, f(x) will be always 0. There is no x ϵ R for which f(x) ϵ (-∞, 0).


It is not bijection.


Question 15.

Mark the correct alternative in each of the following:

Which of the following functions from A = {x : –1 ≤ x ≤ 1} to itself are bijections?

A.
B.

C. h(x) = |x|

D. k(x) = x2


Answer:

Given that A = {x : –1 ≤ x ≤ 1}


a.


It is one-one but not onto.


b.


It is bijective as it is one-one and onto with range [-1, 1].


c. h(x) = |x|


It is not one-one because h(-1)=1 and h(1)=1.


d. k(x) = x2


It is not one-one because k(-1)=1 and k(1)=1.


Question 16.

Mark the correct alternative in each of the following:

Let A = {x : –1 ≤ x ≤ 1} and f : A → A such that f(x) = x |x|, then f is

A. a bijection

B. injective but not surjective

C. surjective but not injective

D. neither injective nor surjective


Answer:

Given that A = {x: –1 ≤ x ≤ 1} and f: A → A such that f(x) = x |x|.


For x <0, f(x) <0


⇒ y = -x2


⇒ x = √-y, which is not possible for x > 0.


Hence, f is one-one and onto.


∴ the given function is bijective.


Question 17.

Mark the correct alternative in each of the following:

If the function f: R → A given by is a surjection, then A =

A. R

B. [0, 1]

C. (0, 1]

D. [0, 1)


Answer:

Given that f: R → A such that is a surjection.


f(x) = y



⇒ y(x2 + 1) = x2


⇒ yx2 + y = x2


⇒ yx2 - x2 = -y




Here,


So, y ϵ [0, 1)


Question 18.

Mark the correct alternative in each of the following:

If a function f : [2, ∞) → B defined by f(x) = x2 – 4x + 5 is a bijection, then B =

A. R

B. [1, ∞)

C. [4, ∞)

D. [5, ∞)


Answer:

Given that a function f : [2, ∞) → B defined by f(x) = x2 – 4x + 5 is a bijection.


Put x = 2 in f(x),


f(x) = 22 – 4× 2 + 5


⇒ f(x=2) = 4 – 8 + 5


⇒ f(x=2) = 1


So, B ϵ [1, ∞)


Question 19.

Mark the correct alternative in each of the following:

The function f : R → R defined by f(x) = (x – 1) (x – 2) (x – 3) is

A. one-one but not onto

B. onto but not one-one

C. both one and onto

D. neither one-one nor onto


Answer:

Given that function f : R → R where f(x) = (x – 1) (x – 2) (x – 3)


If f(x) = f(y)


Then


(x – 1) (x – 2) (x – 3) = (y – 1) (y – 2) (y – 3)


⇒ f(1) = f(2) = f(3) = 0


So, f is not one-one.


y = f(x)


∵ x ϵ R also y ϵ R so f is onto.


Question 20.

Mark the correct alternative in each of the following:

The function f : [–1/2, 1/2] → [π/2, π/2] defined by f(x) = sin–1 (3x – 4x3) is

A. bijection

B. injection but not a surjection

C. surjection but not an injection

D. neither an injection nor a surjection


Answer:

Given that f : [–1/2, 1/2] → [π/2, π/2] where f(x) = sin–1 (3x – 4x3)


Put x = sinθ in f(x) = sin–1 (3x – 4x3)


⇒ f(x=sinθ) = sin–1 (3sinθ – 4sinθ3)


⇒ f(x) = sin–1 (sin3θ)


⇒ f(x) = 3θ


⇒ f(x) = 3 sin–1x


If f(x) = f(y)


Then


3 sin–1x = 3 sin–1y


⇒ x = y


So, f is one-one.


y = 3 sin–1x



∵ x ϵ R also y ϵ R so f is onto.


Hence, f is bijection.


Question 21.

Mark the correct alternative in each of the following:

Let f : R → R be a function defined by Then,

A. f is a bijection

B. f is an injection only

C. f is surjection on only

D. f is neither an injection nor a surjection


Answer:

Given that f: R → R is a function defined as



Here, e|x| is always positive whether x is negative or positive. So, we will get same values of f(x) for different values of x.


Hence, it is not one-one and onto.


∴ f is neither an injection nor a surjection


Question 22.

Mark the correct alternative in each of the following:

Let f : R –{n} → R be a function defined by where m ≠ n. Then,

A. f is one-one onto

B. f is one-one into

C. f is many one onto

D. f is many one into


Answer:

Given that f : R –{n} → R where



Let f(x) = f(y)



⇒ (x-m)(y-n)=(x-n)(y-m)


⇒ xy – xn – my + mn = xy – xm – ny + mn


⇒ x = y


So, f is one-one.




⇒ y(x-n)=(x-m)


⇒ xy – ny = x – m


⇒ x(y-1) = ny – m



For y = 1 , no x is defined.


So, f is into.


Question 23.

Mark the correct alternative in each of the following:

Let f : R → R be a function defined by Then, f is

A. one-one but not onto

B. one-one and onto

C. onto but not one-one

D. neither one-one nor onto


Answer:

Given that f : R → R be a function where



Here, we can see that for negative as well as positive x we will get same value.


So, it is not one-one.


y = f(x)



⇒ y(x2 + 2) = (x2 – 8)


⇒ x2(y-1)=- 2y – 8



For y = 1 , no x is defined.


So, f is not onto.


Question 24.

Mark the correct alternative in each of the following:

f : R → R is defined by is

A. one-one but not onto

B. one-one and onto

C. onto but not one-one

D. neither one-one nor onto


Answer:

Given that f : R → R where



Here, we can see that for negative as well as positive x we will get same value.


So, it is not one-one.


f(x)=y


By definition of onto, each element of y is not mapped to at least one element of x.


So, it is not onto.


Question 25.

Mark the correct alternative in each of the following:

The function f : R → R, f(x) = x2 is

A. injective but not surjective

B. surjective but not injective

C. injective as well as surjective

D. neither injective nor surjective


Answer:

Given that f : R → R, f(x) = x2


Let f(x)=y(x)


⇒ x2 = y2


⇒ x = ±y


So, it is not one-one.


f(x) = y


⇒ x2 = y


⇒ x = ±√y


But co domain is R.


Hence, f is neither injective nor surjective.


Question 26.

Mark the correct alternative in each of the following:

A function f from the set of natural, numbers to the set of integers defined by



A. neither one-one nor onto

B. one-one but not onto

C. onto but not one-one

D. one-one and onto both


Answer:

Given that a function f from the set on natural numbers to integers where



For n is odd


Let f(n) = f(m)



⇒ n = m


For n is even


Let f(n) = f(m)



⇒ n = m


So, f is one-one.


Also, each element of y is associated with at least one element of x, so f is onto.


Hence, f is one-one and onto.


Question 27.

Mark the correct alternative in each of the following:

Which of the following functions from A = {x ϵ R : –1 ≤ x ≤ 1} to itself are bijections?

A. f(x) = |x|

B.

C.

D. none of these


Answer:

Given that A = {x : –1 ≤ x ≤ 1}


a. f(x)=|x|


It is not one-one because f(-1)=1 and f(1)=1.


b.


It is bijective as it is one-one and onto with range [-1, 1].


Question 28.

Mark the correct alternative in each of the following:

Let f: Z → Z be given by Then, f is

A. onto but not one-one

B. one-one but not onto

C. one-one and onto

D. neither one-one nor onto


Answer:

Given function f: Z → Z defined as



For x = 3, f(x) = 0


For x = 5, f(x) = 0


But 3 ≠ 5


So, f is not one-one.


Y=f(x)


∵ x ϵ R ⇒ y ϵ R


∴ Domain = Range


Hence, f is not one-one but onto.


Question 29.

Mark the correct alternative in each of the following:

The function f : R → R defined by f(x) = 6x + 6|x| is

A. one-one and onto

B. many one and onto

C. one-one and into

D. many one and into


Answer:

Given that function f : R → R defined by f(x) = 6x + 6|x|


Let f(x) = f(y)


⇒ 6x + 6|x|=6y + 6|y|


Only when x = y


So, f is one-one.


Now for y=f(x)


y can never be negative which means for no x ϵ R y is negative.


So, f is not onto but into.


Question 30.

Mark the correct alternative in each of the following:

Let f(x) = x2 and g(x) = 2x. Then the solution set of the equation fog(x) = gof (x) is

A. R

B. {0}

C. {0, 2}

D. none of these


Answer:

Given that f(x) = x2 and g(x) = 2x.


Also, fog(x) = gof (x)


⇒ f(2x)=g(x2)



⇒ 2x = x2


⇒ x2 – 2x=0


⇒ x(x-2)=0


⇒ x = 0 or x = 2


Question 31.

Mark the correct alternative in each of the following:

If f: R → R is given by f(x) = 3x – 5, then f–1(x)

A. is given by

B. is given by

C. does not exist because f is not one-one

D. does not exist because f is not onto


Answer:

Given that f: R → R is given by f(x) = 3x – 5


To find f–1(x):


y =f(x)


⇒ y = 3x – 5


⇒ y + 5 = 3x




Question 32.

Mark the correct alternative in each of the following:

If g(f(x)) = |sin x| and f (g(x)) = , then

A. f(x) = sin2x, g(x) = √x

B. f(x) = sin x, g(x) = |x|

C. f(x) = x2, g(x) = sin √x

D. f and g cannot be determined


Answer:

Given that g(f(x)) = |sin x| and


a. For f(x) = sin2x, g(x) = √x


f(g(x))=f(√x)= (sin√x)2


g(f(x))=g(sin2x)= √ sin2x= |sinx|


Correct


b. For f(x) = sin x, g(x) = |x|


f(g(x)) = f(|x|) = sin |x|


g(f(x) = g(sin x) = |sin x|


Incorrect


c. f(x) = x2, g(x) = sin √x


f(g(x)) = f(sin √x) = (sin √x)2


g(f(x)) = g(x2) = sin |x|


Incorrect


Question 33.

Mark the correct alternative in each of the following:

The inverse of the function f : R → [x ϵ R : x < 1] given by is

A.

B.

C.

D. none of these


Answer:

Given that f: R → [x ϵ R : x < 1] defined by



Put y = f(x)




⇒ y(e2x + 1)= e2x – 1


⇒ e2x (y -1) = -y -1



)


)


So, )


Question 34.

Mark the correct alternative in each of the following:

Let A = {x ϵR : x ≥ 1}. The inverse of the function f : A → A given by f(x) = 2x(x–1), is

A.

B.

C.

D. not defined


Answer:

Given that A = {x ϵ R : x ≥ 1}. The function f: A → A given by f(x) = 2x(x–1)


Put y = f(x)


⇒ y = 2x(x–1)









Question 35.

Mark the correct alternative in each of the following:

Let A = {x ϵ R: x ≤ 1} and f : A → A given by f(x) = x(2 – x).Then, f–1(x) is

A.

B.

C.

D.


Answer:

Given that A = {x ϵ R: x ≤ 1} and f : A → A given by f(x) = x(2 – x).


y = f(x)


⇒ y = x(2 – x)


⇒ y = 2x – x2


⇒ y - 1 = 2x – x2 -1


⇒ y – 1 = - (x2 + 1 – 2x)


⇒ (x – 1)2 = 1 – y




Question 36.

Mark the correct alternative in each of the following:

Let Then, {fo(fof)} (x)

A. x for all x ϵ R

B. x for all x ϵ R – {1}

C. x for all x ϵ R –{0, 1}

D. none of these


Answer:

Given that


, for x≠1





, for x≠0




⇒ fofof = x for all x ϵ R –{0, 1}


Question 37.

Mark the correct alternative in each of the following:

If the function f: R → R be such that f(x) = x – [x], where [x] denotes the greatest integer less than or equal to x, then f–1 (x) is

A.

B. [x] – x

C. not defined

D. none of these


Answer:

Given that f: R → R be such that f(x) = x – [x], where [x] denotes the greatest integer less than or equal to x


We will have same value of f for different values of x.


So, the function is not one-one.


∵ f is not bijective


∴ f does not have inverse.


Question 38.

Mark the correct alternative in each of the following:

If F: [1, ∞) → [2, ∞) is given by then f–1(x) equals.

A.

B. ..

C.

D. .


Answer:

Given that F: [1, ∞) → [2, ∞) defined as



y = f(x)




⇒ xy = x2 + 1







Question 39.

Mark the correct alternative in each of the following:

Let g(x) = 1 + x – [x] and where [x] denotes the greatest integer less than or equal to x. Then for all x, f (g(x)) is equal to

A. x

B. 1

C. f(x)

D. g(x)


Answer:

Given that g(x) = 1 + x – [x] and



where [x] denotes the greatest integer less than or equal to x.


(i) -1 < x <0


g(x) = 1 + x – [x]


⇒ g(x) = 1 + x + 1 {∵ [x] = -1}


⇒ g(x) = 2 + x


f(g(x))= f(2 + x)


⇒ f(g(x))=1 + 2 + x-[2 + x]


⇒ f(g(x)) = 3 + x -2 – x


⇒ f(g(x)) = 1


(ii) x = 0


f(g(x)) = f(1 + x-[x])


⇒ f(g(x)) = 1 + 1 + x –[x] – [1 + x + [x]]


⇒ f(g(x)) = 2 + 0 -1


⇒ f(g(x)) = 1


(iii) x > 1


f(g(x)) = f(1 + x-[x])


⇒ f(g(x)) = f(x>0) = 1


Hence, f(g(x)) = 1 for all cases.


Question 40.

Mark the correct alternative in each of the following:

Let Then, for what value of α is f(f(x)) = x?

A. √2

B. –√2

C. 1

D. –1


Answer:

Given that , x≠-1 and f(f(x)) = x





⇒ α2x = x(αx + x + 1)


⇒ α2 = αx + x + 1


⇒ α2 - αx = x + 1


On comparing - αx with x,


We get α = -1


Question 41.

Mark the correct alternative in each of the following:

The distinct linear functions which map [–1, 1] onto [0, 2] are

A. f(x) = x + 1, g(x) = – x + 1

B. f(x) = x – 1, g(x) = x + 1

C. f(x) = –x – 1 g(x) = x – 1

D. none of these


Answer:

a. f(x) = x + 1, g(x) = – x + 1


f(-1) = -1 + 1= 0


f( 1) = 1 + 1 = 2


Also, g(-1) = 1 + 1 = 2


g(1) = -1 + 1= 0


These functions map [–1, 1] onto [0, 2].


b. f(x) = x – 1, g(x) = x + 1


f(-1) = -1 - 1= -2


f( 1) = 1-1 = 0


Also, g(-1) = -1 + 1 = 0


g(1) = 1 + 1= 2


These functions do not map [–1, 1] onto [0, 2].


c. f(x) = –x – 1 g(x) = x – 1


f(-1) = 1 - 1= 0


f( 1) = -1 - 1 = -2


Also, g(-1) = -1 - 1 = -2


g(1) = 1 - 1= 0


These functions do not map [–1, 1] onto [0, 2].


Question 42.

Mark the correct alternative in each of the following:

Let f : [2, ∞) → X be defined by f(x) = 4x – x2. Then, f is invertible, if X =

A. [2, ∞)

B. (–∞, 2]

C. (–∞, 4]

D. [4, ∞)


Answer:

Given that f: [2, ∞) → X be defined by


f(x) = 4x – x2


Let y = f(x)


⇒ y = 4x – x2


⇒ -y + 4 = 4 - 4x + x2


⇒ 4 – y = (x – 2)2




So,


where x < 4


So, x ϵ (–∞, 4]


Question 43.

Mark the correct alternative in each of the following:

If f : R → (–1, 1) is defined by then f–1(x) equals

A.

B.

C.

D. none of these


Answer:

Given that f : R → (–1, 1) is defined by



Here for mod function we will consider three cases, x = 0, x<0 and x>0.


For x< 0




⇒ y(1 + x2) = x2


⇒ x2 (1 – y) = y




Also, checking on x>0 and x = 0 we find that



Question 44.

Mark the correct alternative in each of the following:

Let [x] denote the greatest integer less than or equal to x. If f(x) = sin–1 x, g(x) = [x2] and then

A. fogoh (x) = π/2

B. fogoh (x) = π

C. hofog = hogof

D. hofog ≠ hogof


Answer:

Given that f(x) = sin–1 x, g(x) = [x2] and


a. goh(x) = g(2x)


⇒ goh(x) = [4x2]


fogoh(x) = f([4x2])


⇒ fogoh(x) = sin–1 [4x2]


Hence, given option is incorrect.


b. Similarly, this option is also incorrect.


c. fog(x) = f([x2])


⇒ fog(x) = sin–1 [x2]


hofog(x) = h(sin–1 [x2])


⇒ hofog(x) = 2(sin–1 [x2])


gof(x) = g(sin–1 x)


⇒ gof(x) = [(sin–1 x)2]


hogof(x) = h([(sin–1 x)2])


⇒ hogof(x) = 2[(sin–1 x)2]


Hence, hogof(x) ≠ hofog(x)


Question 45.

Mark the correct alternative in each of the following:

If g(x) = x2 + x – 2 and gof (x) = 2x2 – 5x + 2, then f(x) is equal to

A. 2x – 3

B. 2x + 3

C. 2x2 + 3x + 1

D.2x2 – 3x – 1


Answer:

Given that g(x) = x2 + x – 2 and



a. Let f(x) = 2x – 3


gof(x) = g(2x – 3)


⇒ gof(x) = (2x – 3)2 + 2x - 3 – 2


⇒ gof(x) = 4x2 - 12x + 9 + 2x – 5


⇒ gof(x) = 4x2 - 10x + 4




Hence, this option is the required value of f(x).


b. Let f(x) = 2x + 3


gof(x) = g(2x + 3)


⇒ gof(x) = (2x + 3)2 + 2x + 3 – 2


⇒ gof(x) = 4x2 + 12x + 9 + 2x + 1


⇒ gof(x) = 4x2 + 14x + 10




Hence, this option is not the required value of f(x).


c and d option are incorrect because their degree is more than 1. So, the degree of gof will be more than 2.


Question 46.

Mark the correct alternative in each of the following:

If f(x) = sin2 x and the composite function g(f(x)) = |sin x|, then g(x) is equal to

A.

B.

C.

D.


Answer:

Given that f(x) = sin2 x and the composite function g(f(x)) = |sin x|.


g(f(x)) = g(sin2 x)


a.



Hence, given option is incorrect.


b. If g(x) =√x



⇒ g(f(x)) = |sin x|


Hence, given option is correct.


c.



Hence, given option is incorrect.


d. If g(x) =-√x



⇒ g(f(x)) = - sin x


Hence, given option is incorrect.


Question 47.

Mark the correct alternative in each of the following:

If f : R → R is given by f(x) = x3 + 3, then f–1(x) is equal to

A. x1/3 – 3

B. x1/3 + 3

C.(x – 3)1/3

D. x + 31/3


Answer:

Given that f : R → R is given by f(x) = x3 + 3


Then f–1(x):


y = f(x)


⇒ y = x3 + 3


⇒ y – 3 = x3



So,


Question 48.

Mark the correct alternative in each of the following:

Let f(x) = x3 be a function with domain {0, 1, 2, 3}. Then domain of f–1 is

A. {3, 2, 1, 0}

B. {0, –1, –2, –3}

C. {0, 1, 8, 27}

D. {0, –1, –8, –27}


Answer:

Given that f(x) = x3 be a function with domain {0, 1, 2, 3}.


Then range = {0, 1, 8, 27}


f can be written as {(0, 0), (1, 1), (2, 8), (3, 27)}


f-1 can be written as {(0, 0), (1, 1), (8, 2), (27, 3)}


So, the domain of f–1 is {0, 1, 8, 27}


Question 49.

Mark the correct alternative in each of the following:

Let f : R → R be given by f(x) = x2 – 3. Then, f–1 is given by

A.

B.

C.

D. none of these


Answer:

Given that f : R → R defined by f(x) = x2 – 3


For f–1:


y = f(x)


⇒ y = x2 – 3




Question 50.

Mark the correct alternative in each of the following:

Let f : R → R be given by f(x) = tan x. Then, f–1(1) is

A.

B.

C. does not exist

D. none of these


Answer:

Given that f: R → R be given by f(x) = tan x


For f–1:


y = f(x)


⇒ y = tan x


⇒ x = tan-1 y


f–1 = tan-1 x



Question 51.

Mark the correct alternative in each of the following:

Let f: R → R be defined as

Then, find f(–1) + f(2) + f(4)

A. 9

B. 14

C. 5

D. none of these


Answer:

Given that f: R → R be defined as



For f(-1):


f(x) = 3x


⇒ f(-1) = -3


For f(2):


f(x) = x2


⇒ f(2) = 4


For f(4):


f(x) = 2x


⇒ f(4) = 8


f(–1) + f(2) + f(4) = -3 + 4 + 8


⇒ f(–1) + f(2) + f(4) = 9


Question 52.

Mark the correct alternative in each of the following:

Let A = {1, 2, …, n} and B = {a, b}. Then the number of subjections from A into B is

A. nP2

B. 2n – 2

C. 0

D. none of these


Answer:

Given that A = {1, 2, …, n} and B = {a, b}


The number of functions from a set with n number of elements into a set of 2 number of elements = 2n


But two functions can be many-one into functions.


Hence, answer is 2n – 2.


Question 53.

Mark the correct alternative in each of the following:

If the set A contains 5 elements and the set B contains 6 elements, then the number of one-one and onto mappings from A to B is

A. 720

B. 120

C. 0

D. none of these


Answer:

Given that set A contains 5 elements and set B contains 6 elements.


Number of one-one and onto mappings from A to B means bijections from A to B.


Number of bijections are possible only when n(B) < n(A).


But here, n(A) < n(B)


So, the number of one-one and onto mappings from A to B is 0.


Question 54.

Mark the correct alternative in each of the following:

If the set A contains 7 elements and the set B contains 10 elements, then the number one-one functions from A to B is

A. 10C7

B. 10C7 × 7!

C. 710

D. 107


Answer:

Given that set A contains 7 elements and set B contains 10 elements.


The number one-one functions from A to B is 10C7 × 7!.


Question 55.

Mark the correct alternative in each of the following:

Let be defined by Then,

A. f–1(x) = x

B. f–1(x) = –f(x)

C. fof (x) = x

D.


Answer:

Given that


For f-1:



⇒ y(5x – 3) = 3x + 2


⇒ x ( 5y – 3) = 2 + 3y



So,






⇒ fof(x) = x


Hence, option C is correct.